SlideShare a Scribd company logo
1 of 186
Download to read offline
The category of unemployment arising from the mismatch between the jobs
available in the market and the skills of the available workers in the market
is known as?
A. Frictional Unemployment
B. Cyclic Unemployment
C. Structural Unemployment
D. Disguised Unemployment
E. None of the above
Q.1
Answer: Option C
• Structural changes can be due to change in technology (from labour intensive technology
to capital intensive technology) or change in the pattern of demand.
• Thus, structural unemployment is a category of unemployment arising from the mismatch
between the jobs available in the market and the skills of the available workers in the
market.
Labour Force Participation Rate (LFPR) is defined as the number of
persons in the __________per 1000 persons.
A. Employed persons
B. Unemployed persons
C. Total Population
D. A & B
E. None of the above
Q.2
Answer: Option D
Which one of the following is the correct sequence in the decreasing order of
contribution of different sectors to the Gross Domestic Product of India?
A. Services – Industry – Agriculture
B. Services – Agriculture – Industry
C. Industry – Services – Agriculture
D. Industry – Agriculture – Services
E. Agriculture – Industry – Services
Q.3
Answer: Option A
Research and Development based activity comes under ____________
activity and Retail trade falls under _________ activity.
A. Primary, Quaternary
B. Tertiary, Secondary
C. Quaternary, Tertiary
D. Quaternary, Secondary
E. None of the above
Q.4
Answer: Option C
• Quaternary activities are economic activities that centre around research
and development. This primarily involves creating, organising and interpreting
activities that focus on intellectual development.
➢ Quaternary activities typically centre on research and development and involve
advanced services involving technical skills and specialised knowledge. Some
examples of products and services offered by quaternary activities are robotics, the
internet of things, 3D printing, quantum computing and nanotechnology.
• Retail trade is a tertiary business activity that involves selling goods and
services directly to customers and it occurs through local shops and retail stores
devoted to selling customers. Other selling tactics of retail trading can include
trucks, hand-to-hand cart selling, automatic vending machines, door-to-door
selling, public distribution systems, street peddling and mail order.
• Hence Option C is the answer.
Which of the following phenomenon includes High inflation & high unemployment?
A. Deflation
B. Reflation
C. Disinflation
D. Skew inflation
E. Stagflation
Q.5
Answer: Option E
Reflation is a fiscal or monetary policy designed to expand output, stimulate spending, and curb the effects
of deflation. Methods of reflation are reducing taxes, lowering interest rates, change in money supply, capital
projects etc.
Skewflation is a type of inflation in which the prices of a single commodity or a set of commodities rise while the
overall price level remains stable
Stagflation is a condition in which slow economic growth (stagnation), rising prices (inflation), and rising
unemployment all happen at the same time. Although it is rare for slow economic growth and high inflation to
coexist, it has happened in the past, and many believe it could happen again.
Answer: Option E
Deflation is the economic term used to describe the drop in prices for goods and services. Deflation slows down
economic growth. It normally takes place during times of economic uncertainty when the demand for goods and
services is lower, along with higher levels of unemployment. When prices fall, the inflation rate drops below 0%.
Deflation (and inflation) rates can be calculated using the consumer price index (CPI). This index measures the
changes in the price levels of a basket of goods and services. They can also be measured using the gross domestic
product (GDP) deflator, which measures the price inflation
Disinflation occurs when price inflation slows down temporarily. Unlike deflation, this is not harmful to the
economy because the inflation rate is reduced marginally over a short-term period. Disinflation is the change in the
rate of inflation. Prices do not drop during periods of disinflation and it does not signal an economic slowdown.
Disinflation is demonstrated by a change in the inflation rate from one year to the next. So disinflation would be
measured as a change of 4% from one year to 2.5% in the next.
Which of the following statements is incorrect?
A. The major reason for cyclical unemployment is a lack of demand in the economy and a slowdown in
economic activity.
B. Structural unemployment usually occurs due to the mismatch of skills.
C. Disguised unemployment is when too many people are employed than what is required to produce
efficiently.
D. The minimum amount of unemployment that prevails in an economy due to workers quitting their
previous jobs and are searching for new jobs is called Involuntary Unemployment
E. None of the above
Q.6
Answer: Option D
Cyclical unemployment is due to a deficiency or fall in effective demand from consumers which leads to a fall in production
and low demand for labor.
Structural unemployment refers to a situation that arises due to a change in the structure of the economy. Example:
An economy transforms itself from a Labour-intensive economy to a Capital intensive economy.
Disguised unemployment is when too many people are employed than what is required to produce efficiently.
Involuntary unemployment refers to a situation where workers are seeking work and are willing to work but are unable
to get work.
Hence Option D is the answer.
· The minimum amount of unemployment that prevails in an economy due to workers quitting their previous jobs and are
searching for the new jobs is called Frictional Unemployment.
· Voluntary unemployment refers to a situation where workers are either not seeking for work or are in transition from
one job to another (quitting one job in search of another better job).
· Seasonal unemployment occurs during certain seasons of the year. In some industries and occupations like
agriculture, holiday resorts etc., production activities take place only in some seasons.
What inflation rate RBI is mandated to maintain in the economy with a
margin of 2% on either side?
A. 4%
B. 2%
C. 6%
D. 5%
E. 8%
Q.7
Answer: Option A
In May 2016, the Reserve Bank of India (RBI) Act, 1934 was amended to provide a
statutory basis for the implementation of the flexible inflation targeting
framework.
The amended RBI Act also provides for the inflation target to be set by the
Government of India, in consultation with the Reserve Bank, once in every five
years. Accordingly, the Central Government notified in the Official Gazette 4 per
cent Consumer Price Index (CPI) inflation as the target for the period from August 5,
2016 to March 31, 2021 with the upper tolerance limit of 6 per cent and the lower
tolerance limit of 2 per cent.
On March 31, 2021, the Central Government retained the inflation target and the
tolerance band for the next 5-year period – April 1, 2021 to March 31, 2026.
The National Statistical Organization (NSO) comes under which of the
following Ministries of the Government of India?
A. Ministry of Statistics and Programme Implementation
B. Ministry of Finance
C. Ministry of Commerce and Industry
D. Ministry of Home Affairs
E. Ministry of Labour and Employment
Q.8
Answer: Option A
• The National Statistical Organization (NSO) is headed by a Director
General and comes under the National Statistical Office of the
Ministry of Statistics and Programme Implementation. It is
responsible for conduct of large scale sample surveys in diverse
fields on All India basis.
• Primarily data are collected through nation-wide household
surveys on various socio-economic subjects, Annual Survey of
Industries (ASI), etc.
Arrange the following committees in chronological order in terms of their
formation:
1. Rangarajan Committee
2. Lakdawala Committee
3. Alagh Committee
4. Tendulkar Committee
Which of the statements given above is/are correct?
A. 3-2-4-1
B. 1-4-2-3
C. 1-3-2-4
D. 4-2-1-3
E. 2-1-3-4
Q.9
Answer: Option A
Which of the following is incorrect regarding Wholesale Price Index?
A. It is published by the Office of Economic Adviser, Ministry of Commerce and Industry
B. The base year of it is 2011-12
C. It includes goods only.
D. The highest weightage in the index is given to the primary articles.
E. None of the above
Q.10
Answer: Option D
Wholesale Price Index (WPI)
Wholesale Price Index (WPI) measures the average change in the prices
of commodities for bulk sale at the level of early stage of transactions.
It is published by the Office of Economic Adviser, Ministry of
Commerce and Industry
The base year of it is 2011-12
The index basket of the WPI covers commodities falling under the
three Major Groups namely Primary Articles, Fuel and Power and
Manufactured products.
Answer: Option D
Montreal Protocol aims to achieve which of the following?
A. Protect human health and the environment from anthropogenic emissions and releases of
mercury
B. Protect the ozone layer by phasing out the production of numerous substances that are
responsible for ozone depletion
C. Limit Global warming to the 2 degree Celsius increase from pre industrial level
D. Provide access to Genetic Resources and the Fair and Equitable Sharing of Benefits Arising from
their Utilization
E. None of the above
Q.11
Answer: Option B
The Montreal Protocol on Substances that Deplete the
Ozone Layer is an international treaty designed to protect
the ozone layer by phasing out the production of
numerous substances that are responsible for ozone
depletion.
Which of the following is not a component of India's Forex reserve?
A. Foreign Currency assets
B. Gold reserves
C. Special Drawing Rights (SDRs)
D. India's reserve position in the IMF
E. Foreign Assets with individuals
Q.12
Answer: Option E
• Foreign exchange assets serve many purposes but are most significantly held to
ensure that the central bank has backup funds if the national currency rapidly
devalues or becomes altogether insolvent.
• The four components of forex reserves are foreign currency assets, gold, special
drawing rights and the reserve position in the International Monetary Fund.
Which kind of policy involves raising taxes or cutting government
spending, so that it cuts up on the aggregate demand (thus,
economic growth) to reduce the inflationary pressures in the
economy?
A. Neutral Fiscal policy
B. Contractionary Fiscal policy
C. Expansionary Fiscal Policy
D. Expansionary Monetary Policy
E. Contractionary Monetary Policy
Q.13
Answer: Option B
There has been persistent deficit budget year after year. Which action/actions of
the following can be taken by the Government to reduce the deficit?
1. Reducing revenue expenditure
2. Introducing new Welfare schemes
3. Rationalizing subsidies
4. Reducing import duty
Select the correct answer using the code given below:
A. 1 and 3 only
B. 2 and 3 only
C. 3 and 4 only
D. 1 only
E. 1, 2, 3 and 4
Q.14
Answer: Option A
• Budget deficit is used to define a status of financial health in which expenditures exceed
revenue.
• Introducing new welfare scheme by budgetary support will expand the budget expenditure which
will widen the deficit.
• Similarly, reducing import duty will lead to less revenue for the government, which leads to
increase in budget deficit.
• Rationalizing subsidies and reducing revenue expenditure are two direct ways of reducing the
fiscal burden of the government of India
The term National Income represents____________
A. Gross national product at market prices minus depreciation
B. Gross national product at market prices minus depreciation plus net factor
income from abroad
C. Gross national product at market prices minus depreciation and indirect
taxes plus subsidies
D. Gross national product at market prices minus net factor income from
abroad
E. None of the above
Q.15
Answer: Option C
NNP at FC = gross national product at market prices - depreciation - indirect taxes + subsidies
There will be _________ Budget, if the sum of capital receipts and revenue receipts is greater than the sum
of Capital Expenditure and Revenue Expenditure.
A. Surplus Budget
B. Balanced Budget
C. Revenue Deficit in the Budget
D. Deficit Budget
E. None of the above
Q.16
Answer: Option A
Which of the following best describes the meaning of Fiscal Deficit?
A. The excess of revenue expenditure over revenue receipts
B. The excess of capital expenditure over capital receipts
C. The excess of total expenditure over total receipts excluding borrowings
D. The excess of total expenditure over total receipts
E. None of the above
Q.17
Answer: Option C
Fiscal Deficit - The difference between total revenue and total
expenditure of the government is termed as fiscal deficit. It is an
indication of the total borrowings needed by the government.
While calculating the total revenue, borrowings are not included.
The Sustainable Development Goals or Global Goals are a collection of 17
interlinked global goals designed to be a “blueprint to achieve a better and more
sustainable future for all". These goals are intended to be achieved by which year?
A. 2025
B. 2027
C. 2030
D. 2035
E. 2040
Q.18
Answer: Option C
• The United Nations Sustainable Development Goals (SDGs)
are targets for global development adopted in September
2015, set to be achieved by 2030. All countries of the world
have agreed to work towards achieving these goals.
• Our SDG Tracker presents data across all available indicators
from the Our World in Data database, using official statistics
from the UN and other international organizations. It is a
free, open-access publication that tracks global progress
towards the SDGs and allows people around the world to
hold their governments accountable to achieving the agreed
goals.
• The 17 Sustainable Development Goals are defined in a list
of 169 SDG Targets. Progress towards these Targets is agreed
to be tracked by 232 unique Indicators.
• 2030 Agenda for Sustainable Development Goals which has
been agreed upon by all 193 member states was organised
by United Nations General Assembly
As part of 2030 Agenda for sustainable development, what is the total
number of targets to be achieved under the 17 SDGs?
A. 189
B. 180
C. 179
D. 169
E. 164
Q.19
Answer: Option D
• The United Nations Sustainable Development Goals
(SDGs) are targets for global development adopted
in September 2015, set to be achieved by 2030. All
countries of the world have agreed to work towards
achieving these goals.
• Our SDG Tracker presents data across all available
indicators from the Our World in Data database,
using official statistics from the UN and other
international organizations. It is a free, open-access
publication that tracks global progress towards the
SDGs and allows people around the world to hold
their governments accountable to achieving the
agreed goals.
• The 17 Sustainable Development Goals are defined
in a list of 169 SDG Targets. Progress towards these
Targets is agreed to be tracked by 232 unique
Indicators.
Consider the following statements regarding Human Development Index (HDI):
1. The Human Development Index (HDI) is a composite index that measures the average achievements
in a country in three basic dimensions of human development.
2. The basic dimensions are a long and healthy life, knowledge and a decent standard of living.
3. World Bank releases the index report.
Which of the following statement(s) is/are correct?
A. Only 1 & 2
B. Only 2 &3
C. 1, 2 and 3
D. Only 1
E. Only 2
Q.20
Answer: Option A
• The Human Development Index (HDI) is a summary measure of average achievement in key dimensions of human
development: a long and healthy life, being knowledgeable and having a decent standard of living.
• The Human Development Index (HDI) is a composite index that measures the average achievements in a country in
three basic dimensions of human development.
• These basic dimensions are a long and healthy life, knowledge and a decent standard of living.
Consider the following pairs regarding different sectors of the economy:
1. Primary Sector: It contains all of the economic activities under which the raw materials extracted
are processed and manufactured into final goods.
2. Secondary Sector: The economic activities which take place while exploiting the natural resources
fall under it.
3. Quinary Sector: The highest level of decision-makers in governments and the private corporate
sector fall under it.
4. Quaternary Sector: It is known also as ‘knowledge’ sector, the activities related to education,
research and development, etc. come under it.
Which of the following pairs are correctly matched?
A. 3 only
B. 2 only
C. 1 and 2 only
D. 1, 2 and 3 only
E. 3 & 4 only
Q.21
Answer: Option E
• Primary Sector: The economic activities which take place while exploiting the natural resources fall under it, such
as mining, agricultural activities, oil exploration, etc. When the agriculture sector (one of the sub-sectors of the primary
sector) contributes a minimum of half of the national income and livelihood in a country it is called an agrarian
economy. Hence statement 1 is incorrect.
• Secondary Sector: It contains all of the economic activities under which the raw materials extracted out of the
primary sector are processed (also called the industrial sector). One of its sub-sectors, manufacturing, has proved to
be the largest employer across the western developed Economies. Hence statement 2 is incorrect.
• Tertiary Sector: All of the economic activities where services are produced falls in this sector, such as education,
healthcare, banking, communication, etc. When this sector contributes minimum half of the national income and
livelihood in a country it is called a service economy. Later on, experts created two more sectors of economy—
quaternary and quinary. Though, they are subsectors of the tertiary sector.
• Quaternary Sector: Known also as ‘knowledge’ sector, the activities related to education, research and
development, etc. come under it. The sector plays the most important role in defining the quality of the human
resources an economy has. Hence statement 4 is correct.
• Quinary Sector: All activities where top decisions are made fall under it. The highest level of decision makers in
governments (inclusive of their bureaucracy) and the private corporate sector fall under it. The number of people
involved in this sector is very low rather they are considered the ‘brain’ behind socio-economic performance of an
economy. Hence statement 3 is correct. Therefore, the answer is Option E
Which of the following services is not a part of services sector in India?
A. Transport and Communication
B. Financial Services
C. Defence Services
D. Utility Services
E. Public Administration
Q.22
Answer: Option D
•According to Economic survey, The classification is mentioned in the below table.
"Absence of minimum income to get the minimum needs of life" is
concerned with which of the following types of poverty?
A. Absolute poverty
B. Relative poverty
C. Situational poverty
D. Intergenerational poverty
E. Multidimensional poverty
Q.23
Answer: Option A
Absolute poverty can be defined as the state in which a subject lacks the means to meet his
or her basic needs. Such basic needs are often listed in international poverty reduction
programs, and usually include food, water, shelter, basic education, and basic medical
care. Hence option A is the answer.
Extreme poverty is typically defined as a state in which a person lacks access to all, or
several, of the goods needed for meeting these basic needs.
Relative poverty is closely associated with the issues of inequality. It occurs when people in a
country do not enjoy a certain minimum level of living standards as compared to the rest of
the population and so would vary from country to country, sometimes within the same
country.
Multidimensional poverty is made up of several factors that constitute poor people’s
experience of deprivation – such as poor health, lack of education, inadequate living
standard, lack of income (as one of several factors considered), disempowerment, poor
quality of work and threat from violence.
Mahalanobis Plan Model adopted in India in the Mid-fifties aimed at?
A. Building a strong defence industry base
B. Setting up heavy industries which were capital intensive
C. Curbing inflation in the economy
D. Removing unemployment within a short period
E. None of the above
Q.24
Answer: Option B
• The correct answer is Setting up heavy industries which were capital intensive.
• Mahalanobis Plan Model adopted in India in the Mid-fifties aimed at Setting up heavy
industries which were capital intensive.
➢ In the 1950s, an Old Russian Model was Indianized by P.C. Mahalanobis.
➢ This model is known to have set the statistical foundations for state-directed
investments and created the intellectual underpinnings of the license-raj through
an elaborate input-output model.
➢ This Model suggested that there should be an emphasis on the heavy
industries, which can lead the Indian Economy to a long-term higher growth path.
➢ India's second five-year plan and Industrial policy Resolution 1956, which paved the
way for the development of the Public Sector and license raj.
➢ He was the founder of the Indian Statistical Institute and a close aide of Pandit
Jawahar Lal Nehru.
Which of the following statements is not correct?
A. The first five year plan was launched from 1st April, 1951
B. The first five year plan gave emphasis on Heavy Industrialization
C. The first five year plan was from 1951-1956
D. The first five year plan was based on the Harrod- Domar model
E. None of the above
Q.25
Answer: Option B
• The Second Five year plan emphasized on Heavy industrialization which was based on
P.C. Mahalonobis.
Which of the following is not the correct match?
A. Fiscal Deficit – Excess of Total Expenditure over total receipts less borrowings
B. Budget Deficit – Excess of Total Expenditure over Total Receipts
C. Revenue Deficit – Excess of Total Expenditure over Total revenue receipts
D. Primary Deficit – Excess of total Expenditure over total receipts less borrowings and interest payments
E. None of the Above
Q.26
Answer: Option C
________ is the change in costs of goods and services but does not include those from
the food and energy sectors.
A. Retail Inflation
B. Hyper Inflation
C. Core Inflation
D. Stagflation
E. None of the above
Q.27
Answer: Option C
Core inflation is the change in costs of goods and services, but
does not include those from the food and energy sectors. This
measure of inflation excludes these items because their prices are
much more volatile.
Headline inflation is a measure of the total inflation within an
economy, including commodities such as food and energy prices
(e.g., oil and gas), which tend to be much more volatile and prone
to inflationary spikes
In India, CPI (Rural/Urban/Combined) is published by the _______ and CPI (IW/AL) is
published by Labour Bureau in the Ministry of Labour and Employment.
A. Ministry of Statistics and Programme Implementation
B. Ministry of Corporate Affairs
C. Ministry of Finance
D. Ministry of Home Affairs
E. Reserve Bank of India (RBI)
Q.28
Consumer Price Index
• It measures price changes from the perspective of a retail buyer. It is released by
the National Statistical Office (NSO).
• The CPI calculates the difference in the price of commodities and services such as food,
medical care, education, electronics etc, which Indian consumers buy for use.
• The CPI has several sub-groups including food and beverages, fuel and light, housing and
clothing, bedding and footwear.
• Four types of CPI are as follows:
➢ CPI for Industrial Workers (IW)
➢ CPI for Agricultural Labourer (AL)
➢ CPI for Rural Labourer (RL)
➢ CPI (Rural/Urban/Combined)
Of these, the first three are compiled by the Labour Bureau in the Ministry of Labour
and Employment. Fourth is compiled by the NSO in the Ministry of Statistics and
Programme Implementation.
• Base Year for CPI is 2012
Answer: Option A
Which type of fiscal policy is being described in the following lines?
This policy is designed to boost the economy. It is mostly used in times of high
unemployment and recession. It leads to the government lowering taxes and
spending more, or one of the two. The aim is to stimulate the economy and ensure
consumers' purchasing power does not weaken.
A. Neutral Fiscal policy
B. Contractionary Fiscal policy
C. Expansionary Fiscal Policy
D. Information inadequate
E. None of the Aboveand
Q.38
Expansionary fiscal policy: This policy is designed to boost the economy. It is mostly used in times of
high unemployment and recession. It leads to the government lowering taxes and spending more or
one of the two. The aim is to stimulate the economy and ensure consumers' purchasing power does
not weaken. Hence Option C is the correct answer.
There are two key tools of the fiscal policy:
•Taxation: Funds in the form of direct and indirect taxes, capital gains from investment, etc, help the
government function. Taxes affect the consumer's income and changes in consumption lead to
changes in real gross domestic product (GDP).
•Government spending: It includes welfare programmes, government salaries, subsidies,
infrastructure, etc. Government spending has the power to raise or lower real GDP, hence it is
included as a fiscal policy tool.
Answer: Option C
The Dependency Ratio in India is declining because
A. Population of 0-14 years is relatively high
B. Population of 64 years and above is relatively high
C. Population of 15-64 years is relatively high
D. The population of 0-14 years and 64 years and above together are relatively high
E. Population of 15-64 is relatively low.
Q.39
Key Points
•The Dependency Ratio calculates the number of young and old people and divides them by
the total population including working adults.
•When the dependency ratio is high, it suggests that there is a large elderly/youthful
population.
•The higher the dependency ratio, the more the working-age population has to pay to
sustain its dependents – unless services or social payments are reduced.
•When the dependency ratio is low and working age population is high, it suggests that
there is a working age population. Hence option C is the answer.
Age dependency ratio is the ratio of dependents--people younger than 15 or older than 64--
to the working-age population--those ages 15-64.
Answer: Option C
Which of the following is/are correct about gender budgeting?
1. It is a tool for gender mainstreaming.
2. Gender budgeting seeks to create a separate budget.
3. Gender budgeting recognizes that if gender disparities are to be addressed, adequate monetary
allocations are to be made for the achievement of desired outcomes
Select the correct answer using the code given below:
A. 1 and 2 only
B. 2 and 3 only
C. 1 and 3 only
D. 1, 2 and 3 only
E. 1 only
Q.40
Gender Budgeting (GB)
1. GB is concerned with gender sensitive formulation of legislation, programmes and schemes;
allocation of resources; implementation and execution; audit and impact assessment of
programmes and schemes; and follow-up corrective action to address gender disparities.
2. A powerful tool for achieving gender mainstreaming so as to ensure that benefits of development
reach women as much as men.
3. Does not seek to create a separate budget but seeks affirmative action to address specific needs
of women.
4. Monitors expenditure and public service delivery from a gender perspective.
5. Entails dissection of the Government budgets to establish its gender differential impacts and to
ensure that gender commitments are translated in to budgetary commitments.
6. It is a tool for gender mainstreaming.
7. Gender budgeting uses the budget as an entry point to apply a gender lens to the entire policy
process.
8. Gender budgeting also recognizes that if gender disparities are to be addressed, adequate
monetary allocations are to be made for the achievement of desired outcomes
Answer: Option C
Which of the following is/are the part of the Liquid Adjustment Facility (LAF)?
1.Repo Rate
2.Marginal Standing Facility
3.Bank Rate
4.Standing Deposit Facility
Choose the correct answer using the codes given below:
A. 1 only
B. 2 & 3
C. 1, 2 and 4
D. 1, 2 and 3
E. None of the above
Q.41
Answer: Option C
The Global Financial Stability Report provides an assessment of the global
financial system and markets, and addresses emerging market financing in a
global context. It focuses on current market conditions, highlighting systemic
issues that could pose a risk to financial stability and sustained market access
by emerging market borrowers. The report is published by__________________.
A. International Monetary Fund
B. United Nations Conference on Trade and Development
C. World Economic Forum
D. World Bank
E. UNESCO
Q.42
• The Global Financial Stability Report provides an assessment of the global financial system and markets, and
addresses emerging market financing in a global context. It focuses on current market conditions, highlighting
systemic issues that could pose a risk to financial stability and sustained market access by emerging market
borrowers. The Report draws out the financial ramifications of economic imbalances highlighted by the IMF's
World Economic Outlook. It contains, as special features, analytical chapters or essays on structural or systemic
issues relevant to international financial stability.
• The Global Financial Stability Report (GFSR) is a survey by the IMF staff published twice a year, in the spring and
fall.
• The World Economic Outlook (WEO) and the Global Financial Stability Report (GFSR) are the reports published by
the IMF.
• Global Gender Gap Report, Global Risk Report, and the Travel and Tourism Competitiveness Report are the reports
published by WEF.
• The World Investment Report is released by the United Nations Conference on Trade and Development.
• Global Economic Prospects, Ease of Doing Business, World Development Report are released by the World Bank.
Answer: Option A
“Fiscal Stimulus” is provided to different sectors of an economy to promote the
growth. Which of the following measure does not constitute fiscal stimulus?
A. Input Subsidies
B. Monetary incentives
C. Export subsidies
D. Increasing Taxes
E. Schemes for low interest rate
Q.43
A stimulus package is a number of incentives and tax rebates offered by a government to
boost spending in a bid to pull a country out of a recession or to prevent an economic
slowdown.
• A stimulus package can either be in the form of a monetary stimulus or a fiscal
stimulus.
• A monetary stimulus involves cutting interest rates to stimulate the economy.
• When interest rates are cut, there is more incentive for people to borrow as the cost of
borrowing is reduced.
• An increase in borrowing means there’ll be more money in circulation, less incentive to
save, and more incentive to spend.
• Lowering interest rates could also weaken the exchange rate of a country, thereby
leading to a boost in exports.
• When exports are increased, more money enters the economy, encouraging spending
and stirring up the economy
Answer: Option D
Demographic Transition is described as
A. a stage in which the population growth of the country is zero
B. a process where there is a decline in the birth-rate
C. A long-term trend of declining birth and death rates
D. a process where there is a decline in the death rate
E. a process where there is stagnancy in the society's population
Q.44
•Demographic Transition is a process of change in a society's population overtime. It is a long-term trend of declining birth
and death rates, resulting in substantive change in the age distribution of a population. Hence option C is the answer.
•It can be used to describe and predict the future population of any area.
•It shows changes in birth rate and death rate and consequently on the growth rate of population.
•It is the relationship between economic development and population growth.
•Demographer Warren Thompson first introduced the demographic transition model in 1929.
Answer: Option C
With reference to Gross Value Added (GVA), which of the following statements is/are correct?
A. It is defined as the value of output minus the value of intermediate consumption.
B. Its estimate is released by National Statistical Office.
C. From an economics perspective, it represents the supply side.
D. In India, GVA is calculated at basic prices.
E. All of the above
Q.45
Answer: Option E
Gross Value Added
• In 2015, India opted to make major changes to its compilation of national accounts and decided to bring the whole process into
conformity with the United Nations System of National Accounts (SNA) of 2008.
• The SNA is the internationally agreed standard set of recommendations on how to compile measures of economic activity.
• It describes a coherent, consistent and integrated set of macroeconomic accounts in the context of a set of internationally agreed
concepts, definitions, classifications and accounting rules.
• As per the SNA, GVA is defined as the value of output minus the value of intermediate consumption and is a measure of the
contribution to growth made by an individual producer, industry or sector.
• It provides the rupee value for the number of goods and services produced in an economy after deducting the cost of inputs and raw
materials that have gone into the production of those goods and services.
• It can be described as the main entry on the income side of the nation’s accounting balance sheet, and from an economics
perspective represents the supply side.
• Earlier, India had been measuring GVA at ‘factor cost’ till the new methodology was adopted in which GVA at ‘basic prices’ became
the primary measure of economic output.
• GVA at basic prices will include production taxes and exclude production subsidies.
• GVA at factor cost included no taxes and excluded no subsidies.
• The base year has also been shifted to 2011-12 from the earlier 2004-05.
Which of the following term denotes the average income earned by a person in
India?
A. National Income
B. Per capita income
C. Personal income
D. Personal Disposable Income
E. None of the above
Q.46
Answer: Option B
• Per capita income (PCI) or average income measures the average income earned per
person in a given area (city, region, country, etc.) in a specified year. It is calculated by
dividing the area's total income by its total population. Per capita income is national
income divided by population size.
Why is India regarded as a country with a "Demographic Dividend”?
A. Its high population in the age group below 15 years.
B. Its high population in the age group of 15-64 years.
C. Its high population in the age group above 65 years.
D. Its high total population.
E. Its high population in the age group of 20-80 years.
Q.47
Answer: Option B
The demographic dividend is the economic growth potential that can result from shifts in a population's age
structure, mainly when the share of the working-age population (15 to 64) is larger than the non-working-age
share of the population (14 and younger, and 65 and older).
Demographic dividend in India :
• As populations in countries such as China, US, and Japan is getting older, India's population is getting
younger.
• India's working-age population is now increasing because of rapidly declining birth and death rates.
• India's age dependency ratio, the ratio of dependents (children and the elderly) to the working-age
population (14- to 65-year-olds), is expected to only start rising in 2040, as per UN estimates. This
presents a golden opportunity for economic growth.
• However, this growth will depend on those in the working-age population actually working.
• India's labour force participation rate is declining, especially among rural youth (15- to 29-year-olds)
and women.
• For India to harness the power of its favourable demographics, India's labour force needs to be
empowered with the right skills for the modern economy.
Hence Option B is the answer.
The literacy rate in India is measured after which age?
A. 5 years and above
B. 7 years and above
C. 9 years and above
D. 11 years and above
E. 13 years and above
Q.48
Answer: Option B
The literacy rate is the total percentage of the population of an area at a particular time aged seven
years or above who can read and write with understanding. Hence Option B is the answer.
Consider the following statements with respect to World Trade Organisation (WTO):
1. To facilitate the implementation, administration, and operation of a trade agreement
2. To carry out periodic reviews of the trade policies of its member countries
3. To promote international monetary cooperation
Which of the above statement/s is/are correct?
A. 1 and 2
B. 2 only
C. 3 only
D. 1 and 3
E. 2 and 3
Q.49
Answer: Option A
Functions of the World Trade Organisation:
At the heart of the Organisation are the WTO agreements, negotiated and signed by the bulk of the
world's trading nations. The goal is to help producers of goods and services, exporters, and importers
conduct their business. The WTO's overriding objective is to help trade flow smoothly, freely, fairly,
and predictably.
• It shall facilitate the implementation, administration, and operation of the WTO trade
agreements, such as multilateral trade agreements, plurilateral trade agreements.
• It shall monitor national trade policies.
The WTO was founded on certain guiding principles—non-discrimination, free trade, open, fair and
undistorted competition, etc. In addition, it has special concern for developing countries.
It is the function of International Monetary Fund to promote international monetary cooperation.
Therefore, Option A is the correct answer.
Which of the following institutions is not part of the World Bank community?
A. International Bank for Reconstruction and Development (IBRD)
B. International Development Agency (IDA)
C. Bank for International Settlements (BIS)
D. Multilateral Investment Guarantee Agency (MIGA)
E. All of the above are a part of World Bank Community
Q.50
Answer: Option C
The World Bank Group comprises five constituent institutions: the International
Bank for Reconstruction and Development (IBRD), the International Development
Association (IDA), the International Finance Corporation (IFC), the Multilateral
Investment Guarantee Agency (MIGA), and the International Centre for Settlement
of Investment Disputes (ICSID).
Established in 1930, the BIS is owned by 63 central
banks, representing countries from around the
world that together account for about 95% of world
GDP. Its head office is in Basel, Switzerland and it
has two representative offices: in Hong Kong SAR
and in Mexico City, as well as Innovation Hub
Centres around the world.
The BIS's mission is to support central banks'
pursuit of monetary and financial stability through
international cooperation, and to act as a bank for
central banks.
With reference to ‘G20’, which of the following statements is/are correct?
1.The G20 comprises 19 countries and the European Union.
2.The G20 was founded in 1999 after the Asian financial crisis and later was designated the “premier
forum for international economic cooperation”.
3.Since 2011, the G20 Summit is held annually, under the leadership of a rotating Presidency and latest
G20 Summit is held in Saudi Arabia under its presidency.
Select the correct statement/s:
A. Only 1
B. Only 2
C. Only 2 & 3
D. Only 1 & 2
E. Only 1 & 3
Q.51
Answer: Option B
•Introduction The Group of G20 (G20) comprises Argentina, Australia, Brazil, Canada, China, France, Germany, India,
Indonesia, Italy, Japan, Republic of Korea, Mexico, Russia, Saudi Arabia, South Africa, Turkey, United Kingdom, and the
United States along with the European Union and African Union.
•G20 members currently account for more than 80% of world GDP, 75% of global trade, and 60% of the global population.
Origin and Evolution
•The G20 was founded in 1999 after the Asian financial crisis as a forum for the Finance Ministers and Central Bank
Governors to discuss global economic and financial issues. T
•he G20 was later upgraded to the level of Heads of State/Government and was designated the “premier forum for
international economic cooperation”.
•Since 2011, the G20 Summit is held annually, under the leadership of a rotating Presidency. The G20 initially focused largely
on broad macroeconomic policy, but it has since expanded its ambit to include trade, climate change, sustainable
development, energy, environment, climate change, anti-corruption etc.
•India holds the Presidency of the G20 from 1 December 2022 to 30 November 2023. Hence statement
4 is incorrect.
Which of the following is not the function of the International Monetary Fund?
A. The IMF manages the international monetary system for global payments.
B. IMF advises on macroeconomic policies and exchange rates.
C. It assesses and regulates political sectors in member countries.
D. The IMF offers financial assistance for balance of payments discrepancies.
E. Its role includes promoting economic growth and employment within nations.
Q.52
Answer: Option C
Functions of IMF
1. The International Monetary Fund (IMF) plays a vital role in creating and maintaining the
international monetary system, which facilitates international payments between countries.
2. The IMF focuses on advising and monitoring the macroeconomic policies of member countries,
particularly those affecting exchange rates, government budgets, and money and credit
management.
3. As part of its responsibilities, the IMF assesses a country's financial sector and regulatory policies,
as well as structural policies in the macro economy that impact the labor market and employment.
4. In addition to its advisory role, the IMF is authorized to provide financial assistance to nations
facing balance of payments discrepancies.
5. It contributes to economic stability and growth while striving to maintain high levels of
employment within member countries.
Hence option C is the correct answer.
Poverty Gap is defined as _________
A. the ratio by which the mean income of the poor falls below the poverty line.
B. Gap between rich and poor.
C. Gap between developed nation and developing nation.
D. the intensity of poverty in relative terms of different sections of a country.
E. None of the above
Q.53
Answer: Option A
1. According to the World Bank, poverty gap is the mean shortfall from
the poverty line (counting the non-poor as having zero shortfall),
expressed as a percentage of the poverty line. Poverty gap measures
the intensity of poverty. It shows the extent to which individuals on
average fall below the poverty line.
2. The poverty gap is a ratio showing the average shortfall of the total
population from the poverty line—the minimum level of income
required to secure the basic necessities for survival. In other words, it
reflects the intensity of poverty in a nation.
3. The poverty gap reflects the intensity of poverty in a nation, showing
the average shortfall of the total population from the poverty line.
4. The poverty gap is an indicator produced by the World Bank, which
measures poverty by looking at per capita income and consumption in
households.
5. The data is available for 115 countries and is updated semi-annually in
April and September.
6. The poverty gap statistic is most valuable to economists and
government officials for calculating the poverty gap index.
NABARD was established on the recommendation of which committee?
A. B. Sivaramman committee
B. Narasimhan committee
C. Kotak committee
D. Naresh Chandra committee
E. Narayan Murthy committee
Q.54
Answer: Option A
The recognition of the importance of institutional credit in boosting rural economy
by the Government of India led to the inception of a Committee to review the
Arrangements for Institutional Credit for Agriculture and Rural Development
(CRAFICARD). This was established under the able Chairmanship of Shri B.
Sivaraman, former Member of Planning Commission on 30 March 1979.
Based on the Committee’s interim report recommendation, the creation of National
Bank for Agriculture and Rural Development (NABARD) was approved by the
Parliament through Act 61 of 1981.
✓ Established: NABARD was established on 12th July 1982 on the recommendation of Committee to Review the
Arrangements For Institutional Credit for Agriculture and Rural Development (CRAFICARD) which is also known as the
Sivaraman Comittee.
✓ NABARD is the apex organisation related to financing in the agricultural sector.
✓ Headquarter: Mumbai, Maharashtra.
Right to Education 2009 Act provides free and compulsory education to all children
of the age of ________?
A. 5-15 years
B. 6-14 years
C. 7-15 years
D. 5-14 years
E. 6-15 years
Q.55
Answer: Option B
The Right of Children to Free and Compulsory Education Act,
2009 (the Act) provides for free and compulsory education to
all children of the age of six to fourteen years.
Which of the following is the second stage in the Theory of Demographic
Transition?
[A] High and fluctuating birth and death rates will almost neutralize each other
[B] Death rate is decreasing while the birth rate remains constant at a high level
[C] Birth rate as compared to the death rate declines more rapidly. As a result, the
population grows at a diminishing rate
[D] Birth rate is approximately equal to the death rate and there is little population
growth.
[E] None of the above
Q.56
Answer: Option B
As per SECC-2011, what percentage of total households live in rural
India?
[A] 52.37%
[B] 67.37%
[C] 79.37%
[D] 63.37%
[E] 73.37%
Q.57
Answer: Option E
As per SECC-2011, out of the 24.49 crore households in
India, 17.97 crore households is in villages which is 73.37%
of all households in India.
Which amendment act added the part IX to the Constitution of India
named ‘Panchayati Raj’?
[A] 76th Amendment Act
[B] 75th Amendment Act
[C] 73rd Amendment Act
[D] 74th Amendment Act
[E] None of the above
Q.58
Answer: Option C
The Constitution (Seventy-
Third Amendment) Act, 1992,
which came into force w.e.f.
24th April, 1993, inserted
Part IX in the Constitution of
India and accorded
Panchayats a Constitutional
status as institutions of local
self-governance for rural
India.
Neonatal mortality Rate (NMR) is the number of neonatal deaths per
1000 live births. A neonatal death is defined as a death during which
of the following periods of life?
[A] First 28 days of life
[B] First 31 days of life
[C] First 365 days of life
[D] First 12 days of life
[E] First 7 days of life
Q.59
Answer: Option A
Neonatal death is defined as the death of a live born infant,
regardless of gestational age at birth, within the first 28
completed days of life.
The Maternal Mortality Ratio (MMR) is defined as the
number of maternal deaths during a given time period
per 100,000 live births.
The factors (for example, adverse economic conditions caused by
poverty, low productivity, unemployment, exhaustion of natural
resources and natural calamities) that compel or force a person, due to
various reasons, to leave that place and go to some other place are
known as?
[A] Pull factors
[B] Push factors
[C] Push back factors
[D] Social factors
[E] Political factors
Q.60
Answer: Option B
Explanation: Push Factors
The push factors are those that compel or force a person, due
to various reasons, to leave that place and go to some other
place. For example, adverse economic conditions caused by
poverty, low productivity, unemployment, exhaustion of
natural resources and natural calamities may compel people
to leave their native place in search of better economic
opportunities.
Which of the following committee recommended the establishment of
Regional Rural Banks?
[A] Khusro Committee
[B] Narsimham Committee
[C] Dutt Committee
[D] C Rangarajan Committee
[E] Kotak Committee
Q.61
Answer: Option B
RRBs were set up as government-sponsored, regional based rural lending institutions under the
Regional Rural Banks Act, 1976. The RRBs were established as per the recommendations of the
Narasimham Committee to cater to the rural credit needs of the farming and other rural
communities.The Prathama Grameen Bank was the first bank to be established on 02nd
October 1975. The Syndicate Bank became the first commercial bank to sponsor the Prathama
Grameen Bank RRB
Lending to which of the following sectors is not a part of priority sector
lending?
A. Micro, Small and Medium Enterprises
B. Housing for poor
C. Export credit
D. Iron and Steel industry
E. Social Infrastructure
Q.62
Answer: Option D
It is an international agreement linked to the United Nations Framework Convention on Climate Change,
which commits its Parties by setting internationally binding emission reduction targets. Recognizing that
developed countries are principally responsible for the current high levels of GHG emissions in the
atmosphere as a result of more than 150 years of industrial activity, the Protocol places a heavier burden on
developed nations under the principle of "common but differentiated responsibilities.
Which of the following protocol is mentioned in the passage given above?
A. Montreal Protocol
B. Kyoto Protocol
C. Cartagena Protocol
D. Nagoya Protocol
E. None of the above
Q.63
Kyoto Protocol is an international agreement linked to the United Nations Framework Convention on Climate Change, which
commits its Parties by setting internationally binding emission reduction targets. Recognizing that developed countries are
principally responsible for the current high levels of GHG emissions in the atmosphere as a result of more than 150 years of industrial
activity, the Protocol places a heavier burden on developed nations under the principle of "common but differentiated
responsibilities.
“The Protocol was adopted in Japan, on 11 December 1997 and entered into force on 16 February 2005. The detailed rules for the
implementation of the Protocol were adopted at COP 7 in Marrakesh, Morocco, in 2001, and are referred to as the "Marrakesh
Accords." It covered few market based mechanisms (Flexible Market Mechanisms) to reduce the emissions.
•Most nations have ratified the treaty. The USA is a notable exception to this. It takes the stand that having binding targets only for
developed countries and not polluting countries like China and India is potentially harmful for its own economy. Canada withdrew from
the Kyoto Protocol in 2012.
•The targets are for the following greenhouse gases/gas groups: carbon dioxide, nitrous oxide, methane, sulphur hexafluoride,
hydrofluorocarbons and perfluorocarbons.
•The first commitment period for the agreement was from 2008 to 2012.
•Apart from national measures, the agreement has three mechanisms that are means to achieve the Kyoto targets:
• International Emissions Trading
• Clean Development Mechanism
• Joint Implementation
Answer: Option B
Clean Development Mechanism and Joint implementation are associated with:
A. Kyoto Protocol
B. Cartegena Protocol
C. Nagoya Protocol
D. Montreal Protocol
E. None of the Above
Q.64
Answer: Option A
Answer: Option A
The Kyoto Protocol was adopted in Kyoto, Japan, on 11
December 1997 and entered into force on 16 February 2005.
The Kyoto Protocol was an international treaty which
extended the 1992 United Nations Framework Convention on
Climate Change that commits state parties to reduce
greenhouse gas emissions, based on the scientific consensus
that global warming is occurring and that human-made CO₂
emissions are driving it.
Which of the following committee/s is/are associated with poverty
estimation in India?
[A] Kothari Committee of 1974
[B] Narasimham Committee of 1991
[C] Alagh Committee of 1979
[D] Goswami Committee of 1990
[E] None of the above
Q.65
Answer: Option C
Explanation: Alagh Committee was a task force constituted
by the Planning Commission under the chairmanship of YK
Alagh. It recommended and constructed a poverty line for
rural and urban areas on the basis of nutritional
requirements and related consumption expenditure.
Kothari Committee – UPSC Reforms
Narsimham Committee – Banking Reforms
Alagh Committee – Poverty Estimation
Goswami Committee – Electoral Reforms
No Poverty is a part of which Sustainable Development Goal (SDG)?
[A] SDG 5
[B] SDG 6
[C] SDG 7
[D] SDG 13
[E] SDG 1
Q.66
Answer: Option E
In India, The Child Sex Ratio is calculated based on one of the following
age groups?
[A] 0-6 years
[B] 1-5 years
[C] 0-4 years
[D] 0-3 years
[E] 1-3 years
Q.67
Answer: Option A
Explanation: Sex ratio is calculated as the number of
females per thousand males. The census of child sex ratio in
the age group of zero to six years is conducted every ten years
in India. As per 2011 Census, child sex ratio has shown a
decline from 927 females per thousand males in 2001 to 919
females per thousand males in 2011.
Which of the following Article of the Indian Constitution prohibits
employment of children in factories?
[A] 23
[B] 22
[C] 24
[D] 21
[E] 26
Q.68
Answer: Option C
Article 24 prohibits the
employment of children
below the age of 14 years
in any factory, mine or
other hazardous activities
like construction work or
railway. It is an absolute
prohibition.
How many languages are there in the Eighth Schedule of the Indian
Constitution?
[A] 23
[B] 24
[C] 18
[D] 22
[E] 16
Q.69
Answer: Option D
Explanation: The Eighth Schedule to the
Constitution consists of the following 22
languages: Assamese, Bengali, Gujarati, Hindi,
Kannada, Kashmiri, Konkani, Malayalam,
Manipuri, Marathi, Nepali, Oriya,
Punjabi,Sanskrit, Sindhi, Tamil, Telugu, Urdu,
Bodo, Santhali, Maithili and Dogri.
The total fertility rate (TFR) is
[A] the total number of children born in a country in a given year divided by
labour force
[B] the number of children expected to be born per woman during her
reproductive years
[C] the number of births in a country divided by total population in a given year
[D] the number of women aged 15-45 in a country divided by total population
[E] None of the above
Q.70
Answer: Option B
TFR indicates the average number of children expected to be
born to a woman during her reproductive span of 15-49
years.
The Total Fertility Rate (TFR) is a standard demographic
indicator used internationally to estimate the average
number of children that a woman would have over her
childbearing years (i.e. age 15-49), based on current birth
trends.
What are the ideals that a state must keep in mind while framing
policies called?
A. Fundamental Rights
B. Directive Principles of State Policy
C. Gandhian Principles
D. Socialism
E. None of the above
Q.71
Answer: Option B
DPSP are the ideals that must be kept in mind by the state
whenever it formulates policies and enacts laws.
CONSTITUTIONAL PROVISIONS
Directive Principles of State Policy are enumerated
in Part IV of the Constitution from Articles 36 to
51.
• Constitution of Ireland (1937) was the source
for the Directive principles in the Indian
Constitution.
• These Principles resemble the ‘Instrument of
Instructions’ enumerated in the Government of
India Act of 1935.
Principles of DPSP are borrowed from which constitution?
A. The USA Constitution
B. Canadian Constitution
C. UK Constitution
D. Irish Constitution
E. None of the above
Answer:- [D]
Irish Constitution of 1937
Q.72
Who among the following is associated with Beej Bachao Andolan
initiated in 1986?
[A] Sunderlal Bahuguna
[B] Pandurang Hegde
[C] Medha Patkar
[D] Vijay Jardhari
[E] None of the above
Q.73
Answer: Option D
The 'Beej Bachao Andolan', also called, 'Save the Seed
Movement' was started by the farmers with the help of an
activist named Vijay Jardhari in Uttarakhand in the early
1980s. It was exactly stated in 1986.
Which article states about promotion of the educational and economic
interests of SC, ST, and other the weaker sections of the people under
the Directive Principles of State Policy?
A. Article 46
B. Article 44
C. Article 42
D. Article 49
E. None of the above
Q.74
Answer: Option A
Which amendment made elementary education for all children a
fundamental right? Also choose the article added after that:
A. 80th Amendment, Article 21
B. 86th Amendment, Article 21
C. 44th Amendment, Article 22 A
D. 86th Amendment, Article 21 A
E. None of the above
Q.75
Answer: Option D
Explanation: The 86th Amendment Act of 2002 changed the
subject-matter of Article 45 and made elementary education
a fundamental right under Article 21 A.
Which part of the Indian Constitution contains the Fundamental
Rights?
A. Part I
B. Part II
C. Part III
D. Part IV
E. None of the above
Q.76
Answer: Option C
Explanation: Article 12 to 35 contained in Part III of the
Constitution deal with Fundamental Rights. These are: Right
to equality, including equality before law, prohibition of
discrimination on grounds of religion, race, caste, sex or
place of birth, and equality of opportunity in matters of
employment.
Article 32 of the Indian Constitution guarantees:
A. Right to Equality
B. Right to Freedom of Religion
C. Right to Constitutional Remedies
D. Right against Exploitation
E. None of the above
Q.77
Answer: Option C
Explanation: Article 32 of the Indian Constitution gives the
right to individuals to move to the Supreme Court to seek
justice when they feel that their right has been 'unduly
deprived'.
The ‘melting pot’ and ‘salad bowl’ are theories related to which of the
following concepts:
A. Migration
B. Demographic Transition
C. Poverty
D. Multiculturalism
E. None of the above
Q.78
Answer: Option D
Explanation: Multiculturalism is a society's way of dealing
with cultural diversity, both at the national level and at the
community level. Multiculturalism develops according to one
of two theories, namely the “melting pot” theory or the “salad
bowl” theory.
Which of the following schedules provides for division of power
between the Centre and States under union list, state list and
concurrent list?
A. 7th Schedule
B. 10th Schedule
C. 12th Schedule
D. 1st Schedule
E. None of the above
Q.79
Answer: Option A
Explanation: The 7th Schedule of Indian Constitution has
three lists namely, the Union list, state list, and concurrent
list that show the division of power between the Union and
States concerning certain subjects. The Union List has a total
of 97 subjects, the State List has 66 subjects, and the
Concurrent List has 47 Subjects.
Which of the following statements correctly describes Balance of Payment
Account?
A. It is a systematic record of some economic transactions of residents of a country with the rest
of the world during a given period of time
B. It is a systematic record of all economic transactions of non-residents of a country with the
residents during a given period of time
C. It is a systematic record of all economic transactions of residents of a country with the rest of
the world during a given period of time
D. It is a systematic record of all economic transactions and non-economic transactions of
residents of a country with the rest of the world during a given period of time
E. None of the Above
Q.80
The balance of payments is a systematic record of all economic transactions of residents of a country with
the rest of the world during a given period of time.
•The balance of payments (BOP), also known as the balance of international payments, summarizes all
transactions that a country’s individuals, companies, and government bodies complete with individuals,
companies, and government bodies outside the country. These transactions consist of imports and exports
of goods, services, and capital, as well as transfer payments, such as foreign aid and remittances. Hence
Option C is the answer.
Answer: Option C
Which of the following is not accounted for in the capital account under Balance of
Payment?
A. Direct Investment
B. External Commercial Borrowing
C. Remittances and Grants
D. Portfolio Investment
E. All of the above are accounted for in the capital account under Balance of Payment
Q.81
Answer: Option C
There are two components of BoP-
1. Current Account
2. Capital Account
Current Account deals with current, ongoing, short term transactions like trade in goods, services (invisible) etc. It reflects the
nation’s net income. It has broadly 4 components.
• Goods – trade in goods
• Services (invisible) – trade in services e.g. tourism
• Income – investment income
• Current unilateral transfers – donations, gifts, grants, remittances .
Note that grants might appear as the component of the capital account but are included in the current account as they are unilateral
and creates no liability. The recipient does not have to give anything back in return.
Option C is incorrect because remittances and grants are part of Current account.
Capital Account deals with capital transactions i.e. those transactions which create assets or liabilities. It reflects the net changes in
the ownership of national assets. It also has broadly 4 components
• Foreign Direct Investment (FDI)
• Foreign Portfolio Investment (FPI)
• External Borrowings such as ECB
• Reserve Account with the Central Bank.
Answer: Option C
All of the following are variables that can be manipulated to affect fiscal policy except:
A. Income taxes
B. Government Spending more on national highways.
C. Paying higher subsidies
D. Paying unemployment insurance benefits.
E. Rate of interest
Q.82
Answer: Option E
• The Indian government coordinates with the RBI. RBI either decreases or increases the money supply which will
influence the rate of interest
• Government as part of its fiscal policy tampers with Tax rates and its expenditure on various programs.
In the context of Balance of payment (BoP), what are ‘accommodating transactions’?
A. Mismatches in Balance of payment records.
B. Current account transactions.
C. Transactions that are done to balance the surplus or deficit of Balance of payment.
D. Transactions that are done with profit maximization motive.
E. None of the Above
Q.83
Accommodating Transactions
•Transactions done to balance the surplus or deficit of BoP caused by the Current account
and Autonomous transactions are called accommodating transactions. Hence option C is
the answer.
These include –
•Foreign exchange reserve
•Borrowing from IMF or foreign monetary authorities
Answer: Option C
Which of the following is not the correct statement relating to the benefits of small-scale industries for an economy?
A. They provide immediate large-scale employment.
B. They offer a method of ensuring a more equitable distribution of the national income.
C. They facilitate an effective mobilisation of resources of capital and skill.
D. They can compete with foreign heavy industries and can provide heavy machineries.
E. They bring about the regional balances in the country.
Q.84
Answer: Option D
• The Micro, Small and Medium Enterprises (MSME) sector has emerged as a highly vibrant and dynamic sector of the
Indian economy over the last five decades. It contributes significantly in the economic and social development of the
country by fostering entrepreneurship and generating large employment opportunities at comparatively lower capital
cost, next only to agriculture. MSMEs are complementary to large industries as ancillary units and this sector
contributes significantly in the inclusive industrial development of the country. The MSMEs are widening their
domain across sectors of the economy, producing diverse range of products and services to meet demands of
domestic as well as global markets.
• As per the Annual report 2022-23 of the Ministry of Micro, Small and Medium Enterprises, the MSME sector
provided employment to 11.10 Crore people. MSMEs sector contributes around 30% to India's GDP and has 40%
share in the country's exports.
_____ has recently joined the Shanghai Cooperation Organisation at its 23rd leaders’ summit,
hosted virtually by India as chairman of the grouping.
(a) Pakistan
(b) Iran
(c) Russia
(d) Kazakhstan
(e) Uzbekistan
Q.85
Answer: Option B
Which of the following statements are incorrect regarding BRICS?
(a) BRICS does not exist in form of organization, but it is an annual summit
(b) BRICS started as BRIC and Spain was invited later to join.
(c) BRICS bloc of developing nations agreed to admit Saudi Arabia, Iran, Ethiopia, Egypt, Argentina
and the United Arab Emirates
(d) B and C
(e) A and C
Q.86
Answer: Option B
Announcing the decision at the end of the 15th BRICS summit at
Johannesburg, Prime Minister Narendra Modi said addition of new
members would strengthen the outfit and increase confidence in the idea
of multipolar world order.
Q. Which of the following ministry in India decides the rate of depreciation of a good?
(a) Ministry of Finance
(b) Ministry of Commerce and Industry
(c) Ministry of Statistics and Programme implementation
(d) Ministry of Railways
(e) Ministry of defense
Q.87
Every asset (except human beings) goes for depreciation in the process of their use, which
means they ‘wear and tear’.
The governments of the economies decide and announce the rates by which assets
depreciate (done in India by the Ministry of Commerce and Industry) and a list is published,
which is used by different sections of the economy to determine the real levels of
depreciation in different assets. Hence option B is the answer.
Answer: Option B
In terms of economy, the Visit by foreign nationals to witness the 2023 Cricket World Cup in
India would amount to
(a) Export
(b) Import
(c) Production
(d) Consumption
(e) Investment
Q.88
Exports are goods and services that are produced in one country and sold to buyers in another. The
visit of foreign nationals to India amounted to export as foreign nationals will buy Indian goods and
use Indian services. Also, India will get foreign currency. Hence Option A is the answer. An import is a
good or service bought in one country that was produced in another. Suppose a citizen of the US buys
goods and services in India, then it would amount to Import for the US. Because its citizen is buying
goods and services from another country. Production - The process of making or growing goods to be
sold. Consumption - The amount used or eaten. Investment - An investment is an asset or item
acquired with the goal of generating income.
Answer: Option A
Q. The dominant objective of Monetary Policy is
(a) Controlling Money Supply
(b) Controlling Demand for Money
(c) Achieving Price Stability
(d) Promoting Foreign Trade
(e) Reduction of Unemployment
Q.89
Explanation: Monetary Policy refers to the use of monetary instruments under the control of the
central bank to influence variables, such as interest rates, money supply, and availability of credit, to
achieve the objectives of the policy.
Monetary Policy could have either a single objective of price stability or multiple objectives of the
policy. In theory and practice, price stability is considered the dominant objective of monetary policy.
For countries, which have adopted an inflation-targeting framework price stability is the core
objective. Therefore, the correct answer is 'Achieving Price stability'. Hence option C is the correct
answer.
Answer: Option C
Q. Which of the following statement is NOT correct regarding the New Development
Bank (NDB)?
(a) It is established by the BRICS countries
(b) It is headquartered at Sanghai, China
(c) K.V Kamath was the first president of the Bank
(d) The initial authorised capital of the bank was $100 bn
(e) It cannot lend to countries other than BRICS members
Answer: E
Q.90
Q. It is a type of poverty generally caused by a sudden crisis or loss and is often temporary.
It can be caused by environmental disasters, divorce, or severe health problems. Which kind
of poverty are we referring to?
(a) Generational poverty
(b) Situational poverty
(c) Absolute poverty
(d) Relative poverty
(e) None of the above
Answer: B
Q.91
Q. When we say that a person has more money in his hand to save or to consume when
compared to the previous year for example, then there is a definite increase in which of the
following?
(a) Portion of National Income received by individuals
(b) Personal Disposable Income of that person
(c) Private Income
(d) Personal Income
(e) None of the above
Answer: B
Q.92
Q. Which of the prices comes into play during the billing process, where the indirect taxes
levied by the government are added and this is the final cost the consumer has to pay for
the product and the subsidies are subtracted because it reduces the cost of production and
thus the consumer has to pay less.
(a) Factor Cost
(b) Basic Price
(c) Market Prices
(d) Current Prices
(e) None of the above
Answer: C
Q.93
Q. A country wants to know whether the quantity of goods produced in that country in a
year has increased when compared to the previous year. Which of the following would give
an idea regarding the same?
(a) Real GDP
(b) Nominal GDP
(c) Per Capita GDP
(d) Purchasing Power Parity
(e) None of the Above
Q.94
GDP can be calculated at:
a. Current prices (Nominal GDP)
b. Constant prices (Real GDP)
Nominal GDP: Nominal GDP refers to the current year's production of final goods and services valued
at current year's prices.
Real GDP: If this measure is adjusted for inflation; it is expressed in real terms. Real GDP refers to the
current year production of goods and services valued at base year prices. Base year prices are constant
prices.
Answer: Option A
Which one of the following is not a revenue receipt in government budget?
A. Direct taxes
B. Recovery of loans
C. Profits and Dividends
D. Fees, Penalty and Fine
E. Interest receipts
Q.95
Answer: Option B
Which one of the following is a capital receipt in government budget?
A. Interest receipts on loans given by the government to other parties
B. Dividends and profits from public sector undertakings
C. Borrowing of the government from public
D. Income tax receipts
E. Fines
Q.96
Answer: Option C
• Capital receipts are receipts that create liabilities or reduce financial assets. They also refer to incoming cash flows.
• Capital receipts can be both non-debt and debt receipts.
• Loans from the general public, foreign governments and the Reserve Bank of India (RBI) form a crucial part of capital
receipts.
Which of the following statement(s) is/are correct regarding capital goods?
1. These are final goods and not the intermediate goods.
2. They themselves do not get transformed in the production process.
3. Over a period of time, the monetary value of most of the capital goods gets increased.
4. Capital goods are one of the factors of production.
A. 1 and 3 only
B. 2 and 3 only
C. 1, 2 and 4 only
D. 1, 2 and 3 only
E. All of the above
Q.97
Answer: Option C
• Capital goods are tangible assets such as buildings, machinery, equipment, vehicles and
tools that an organization uses to produce goods or services in order to produce consumer
goods and goods for other businesses.
• Manufacturers of automobiles, aircraft, and machinery fall within the capital goods
sector because their products are used by companies involved in manufacturing, shipping and
providing other services.
• Statement 3 is incorrect as during the course of time, the value of the capital goods gets
decreased.
• This concept is known as depreciation.
• Depreciation: The capital gets consumed during the year due to wear and tear. This wear and
tear is called depreciation. E.g. We buy a new car for Rs.5,00,000. As the years pass by, the car
gets used and needs maintenance. So after say for 5 years, the value of the car does not
remain the same.
• The other factors of production are Land, Labour and Entrepreneurship.
Which of the following isare not the Qualitative Tools of Monetary Policy?
A. Credit Rationing
B. Moral Suasion
C. Cash reserve Ratio
D. Direct Action
E. All of the Above are the Qualitative Tools of Monetary Policy
Q.98
Answer: Option C
• Qualitative tools are direct and specific in nature. Qualitative
tools include persuasion by the Central bank in order to make
commercial banks discourage or encourage lending which is done
through moral suasion.
• Qualitative also called selective credit control instruments work
through regulation of margin requirement, credit rationing,
regulation of consumer credit, moral suasion and direct action.
Standing Deposit is a tool used as part of the Liquidity Adjustment Facility. Which of the following statements
is wrong for SDF?
A. Standing Deposit Facility can be used to suck liquidity
B. Standing Deposit Facility need Government Securities​
C. Standing Deposit Facility changes with Repo Rate
D. Under Standing Deposit Facility, banks can deposit money with RBI
E. None of the Above
Q.99
Answer: Option B
In the context of any county, which one of the following would be considered as part of its social capital?
A. The population of the country
B. The stock of its buildings, other infrastructure and machines
C. The size of population in the working age group
D. The level of mutual trust and harmony in the society
E. None of the above
Q.100
Answer: Option D
• The level of mutual trust and harmony in the society.
• The stock of its buildings, other infrastructure, and machines implies a physical
capital.
• Social capital is the values, beliefs, and attitudes that govern the nature of social
interactions.
• Social capital refers to connections among individuals — social networks and the
norms of reciprocity and trustworthiness that arise from them i.e. the level of mutual
trust and harmony in the society.
ECONOMICS AND SOCIAL ISSUES PDF FOR RBI GRADE B EXAM
ECONOMICS AND SOCIAL ISSUES PDF FOR RBI GRADE B EXAM

More Related Content

Similar to ECONOMICS AND SOCIAL ISSUES PDF FOR RBI GRADE B EXAM

Quiz, week #2Measuring macro outcomesMy expectations are that .docx
Quiz, week #2Measuring macro outcomesMy expectations are that .docxQuiz, week #2Measuring macro outcomesMy expectations are that .docx
Quiz, week #2Measuring macro outcomesMy expectations are that .docxcatheryncouper
 
Fundamental and technical analysis
Fundamental and technical analysisFundamental and technical analysis
Fundamental and technical analysisAashishKhatkar2
 
Macroeconomic variables Assignment
Macroeconomic variables Assignment Macroeconomic variables Assignment
Macroeconomic variables Assignment Ali Shah
 
money and banking.pptx
money and banking.pptxmoney and banking.pptx
money and banking.pptxssuserdd894c
 
"India in search of a way to harness the Inflation dragon" case study of Macr...
"India in search of a way to harness the Inflation dragon" case study of Macr..."India in search of a way to harness the Inflation dragon" case study of Macr...
"India in search of a way to harness the Inflation dragon" case study of Macr...Nikhil Gupta
 
Demand-Management Policies.pptx
Demand-Management Policies.pptxDemand-Management Policies.pptx
Demand-Management Policies.pptxANTHONYAKINYOSOYE1
 
Macro Economics PPT
Macro Economics PPT Macro Economics PPT
Macro Economics PPT M Asif Bhat
 
Ch13 econ.ppt
Ch13 econ.pptCh13 econ.ppt
Ch13 econ.pptsyedmohd9
 
Intro to Macroeconomics
Intro to MacroeconomicsIntro to Macroeconomics
Intro to Macroeconomicsmscuttle
 
Intro to Macroeconomics
Intro to MacroeconomicsIntro to Macroeconomics
Intro to Macroeconomicsmscuttle
 
Rana alyousef Macro H.W 1 miss miryam College of .docx
Rana alyousef Macro H.W  1 miss miryam College of .docxRana alyousef Macro H.W  1 miss miryam College of .docx
Rana alyousef Macro H.W 1 miss miryam College of .docxmakdul
 
“Analysis of GDP, Unemployment and Inflation rates using mathematical formula...
“Analysis of GDP, Unemployment and Inflation rates using mathematical formula...“Analysis of GDP, Unemployment and Inflation rates using mathematical formula...
“Analysis of GDP, Unemployment and Inflation rates using mathematical formula...IRJET Journal
 
PLEASE REWORD THESE PARAGRAPHS IN YOUR OWN WORDS. PLEASE DO NOT US.docx
PLEASE REWORD THESE PARAGRAPHS IN YOUR OWN WORDS. PLEASE DO NOT US.docxPLEASE REWORD THESE PARAGRAPHS IN YOUR OWN WORDS. PLEASE DO NOT US.docx
PLEASE REWORD THESE PARAGRAPHS IN YOUR OWN WORDS. PLEASE DO NOT US.docxLeilaniPoolsy
 
Macroeconomic environment ppt
Macroeconomic environment ppt Macroeconomic environment ppt
Macroeconomic environment ppt Babasab Patil
 

Similar to ECONOMICS AND SOCIAL ISSUES PDF FOR RBI GRADE B EXAM (20)

Inflation
InflationInflation
Inflation
 
Types of inflation
Types of inflationTypes of inflation
Types of inflation
 
Quiz, week #2Measuring macro outcomesMy expectations are that .docx
Quiz, week #2Measuring macro outcomesMy expectations are that .docxQuiz, week #2Measuring macro outcomesMy expectations are that .docx
Quiz, week #2Measuring macro outcomesMy expectations are that .docx
 
Fundamental and technical analysis
Fundamental and technical analysisFundamental and technical analysis
Fundamental and technical analysis
 
Macroeconomic variables Assignment
Macroeconomic variables Assignment Macroeconomic variables Assignment
Macroeconomic variables Assignment
 
Capstone
CapstoneCapstone
Capstone
 
money and banking.pptx
money and banking.pptxmoney and banking.pptx
money and banking.pptx
 
What is Economy
What is EconomyWhat is Economy
What is Economy
 
"India in search of a way to harness the Inflation dragon" case study of Macr...
"India in search of a way to harness the Inflation dragon" case study of Macr..."India in search of a way to harness the Inflation dragon" case study of Macr...
"India in search of a way to harness the Inflation dragon" case study of Macr...
 
macro economic concepts
macro economic conceptsmacro economic concepts
macro economic concepts
 
Inflation in india
Inflation in indiaInflation in india
Inflation in india
 
Demand-Management Policies.pptx
Demand-Management Policies.pptxDemand-Management Policies.pptx
Demand-Management Policies.pptx
 
Macro Economics PPT
Macro Economics PPT Macro Economics PPT
Macro Economics PPT
 
Ch13 econ.ppt
Ch13 econ.pptCh13 econ.ppt
Ch13 econ.ppt
 
Intro to Macroeconomics
Intro to MacroeconomicsIntro to Macroeconomics
Intro to Macroeconomics
 
Intro to Macroeconomics
Intro to MacroeconomicsIntro to Macroeconomics
Intro to Macroeconomics
 
Rana alyousef Macro H.W 1 miss miryam College of .docx
Rana alyousef Macro H.W  1 miss miryam College of .docxRana alyousef Macro H.W  1 miss miryam College of .docx
Rana alyousef Macro H.W 1 miss miryam College of .docx
 
“Analysis of GDP, Unemployment and Inflation rates using mathematical formula...
“Analysis of GDP, Unemployment and Inflation rates using mathematical formula...“Analysis of GDP, Unemployment and Inflation rates using mathematical formula...
“Analysis of GDP, Unemployment and Inflation rates using mathematical formula...
 
PLEASE REWORD THESE PARAGRAPHS IN YOUR OWN WORDS. PLEASE DO NOT US.docx
PLEASE REWORD THESE PARAGRAPHS IN YOUR OWN WORDS. PLEASE DO NOT US.docxPLEASE REWORD THESE PARAGRAPHS IN YOUR OWN WORDS. PLEASE DO NOT US.docx
PLEASE REWORD THESE PARAGRAPHS IN YOUR OWN WORDS. PLEASE DO NOT US.docx
 
Macroeconomic environment ppt
Macroeconomic environment ppt Macroeconomic environment ppt
Macroeconomic environment ppt
 

Recently uploaded

Top Rated Pune Call Girls Aundh ⟟ 6297143586 ⟟ Call Me For Genuine Sex Servi...
Top Rated  Pune Call Girls Aundh ⟟ 6297143586 ⟟ Call Me For Genuine Sex Servi...Top Rated  Pune Call Girls Aundh ⟟ 6297143586 ⟟ Call Me For Genuine Sex Servi...
Top Rated Pune Call Girls Aundh ⟟ 6297143586 ⟟ Call Me For Genuine Sex Servi...Call Girls in Nagpur High Profile
 
falcon-invoice-discounting-unlocking-prime-investment-opportunities
falcon-invoice-discounting-unlocking-prime-investment-opportunitiesfalcon-invoice-discounting-unlocking-prime-investment-opportunities
falcon-invoice-discounting-unlocking-prime-investment-opportunitiesFalcon Invoice Discounting
 
Top Rated Pune Call Girls Shikrapur ⟟ 6297143586 ⟟ Call Me For Genuine Sex S...
Top Rated  Pune Call Girls Shikrapur ⟟ 6297143586 ⟟ Call Me For Genuine Sex S...Top Rated  Pune Call Girls Shikrapur ⟟ 6297143586 ⟟ Call Me For Genuine Sex S...
Top Rated Pune Call Girls Shikrapur ⟟ 6297143586 ⟟ Call Me For Genuine Sex S...Call Girls in Nagpur High Profile
 
Call Girls in New Friends Colony Delhi 💯 Call Us 🔝9205541914 🔝( Delhi) Escort...
Call Girls in New Friends Colony Delhi 💯 Call Us 🔝9205541914 🔝( Delhi) Escort...Call Girls in New Friends Colony Delhi 💯 Call Us 🔝9205541914 🔝( Delhi) Escort...
Call Girls in New Friends Colony Delhi 💯 Call Us 🔝9205541914 🔝( Delhi) Escort...Delhi Call girls
 
VIP Call Girl in Mumbai 💧 9920725232 ( Call Me ) Get A New Crush Everyday Wit...
VIP Call Girl in Mumbai 💧 9920725232 ( Call Me ) Get A New Crush Everyday Wit...VIP Call Girl in Mumbai 💧 9920725232 ( Call Me ) Get A New Crush Everyday Wit...
VIP Call Girl in Mumbai 💧 9920725232 ( Call Me ) Get A New Crush Everyday Wit...dipikadinghjn ( Why You Choose Us? ) Escorts
 
Solution Manual for Principles of Corporate Finance 14th Edition by Richard B...
Solution Manual for Principles of Corporate Finance 14th Edition by Richard B...Solution Manual for Principles of Corporate Finance 14th Edition by Richard B...
Solution Manual for Principles of Corporate Finance 14th Edition by Richard B...ssifa0344
 
VIP Call Girl in Mumbai Central 💧 9920725232 ( Call Me ) Get A New Crush Ever...
VIP Call Girl in Mumbai Central 💧 9920725232 ( Call Me ) Get A New Crush Ever...VIP Call Girl in Mumbai Central 💧 9920725232 ( Call Me ) Get A New Crush Ever...
VIP Call Girl in Mumbai Central 💧 9920725232 ( Call Me ) Get A New Crush Ever...dipikadinghjn ( Why You Choose Us? ) Escorts
 
Booking open Available Pune Call Girls Wadgaon Sheri 6297143586 Call Hot Ind...
Booking open Available Pune Call Girls Wadgaon Sheri  6297143586 Call Hot Ind...Booking open Available Pune Call Girls Wadgaon Sheri  6297143586 Call Hot Ind...
Booking open Available Pune Call Girls Wadgaon Sheri 6297143586 Call Hot Ind...Call Girls in Nagpur High Profile
 
VIP Call Girl in Thane 💧 9920725232 ( Call Me ) Get A New Crush Everyday With...
VIP Call Girl in Thane 💧 9920725232 ( Call Me ) Get A New Crush Everyday With...VIP Call Girl in Thane 💧 9920725232 ( Call Me ) Get A New Crush Everyday With...
VIP Call Girl in Thane 💧 9920725232 ( Call Me ) Get A New Crush Everyday With...dipikadinghjn ( Why You Choose Us? ) Escorts
 
VIP Independent Call Girls in Bandra West 🌹 9920725232 ( Call Me ) Mumbai Esc...
VIP Independent Call Girls in Bandra West 🌹 9920725232 ( Call Me ) Mumbai Esc...VIP Independent Call Girls in Bandra West 🌹 9920725232 ( Call Me ) Mumbai Esc...
VIP Independent Call Girls in Bandra West 🌹 9920725232 ( Call Me ) Mumbai Esc...dipikadinghjn ( Why You Choose Us? ) Escorts
 
Kharghar Blowjob Housewife Call Girls NUmber-9833754194-CBD Belapur Internati...
Kharghar Blowjob Housewife Call Girls NUmber-9833754194-CBD Belapur Internati...Kharghar Blowjob Housewife Call Girls NUmber-9833754194-CBD Belapur Internati...
Kharghar Blowjob Housewife Call Girls NUmber-9833754194-CBD Belapur Internati...priyasharma62062
 
VIP Independent Call Girls in Taloja 🌹 9920725232 ( Call Me ) Mumbai Escorts ...
VIP Independent Call Girls in Taloja 🌹 9920725232 ( Call Me ) Mumbai Escorts ...VIP Independent Call Girls in Taloja 🌹 9920725232 ( Call Me ) Mumbai Escorts ...
VIP Independent Call Girls in Taloja 🌹 9920725232 ( Call Me ) Mumbai Escorts ...dipikadinghjn ( Why You Choose Us? ) Escorts
 
Call Girls Banaswadi Just Call 👗 7737669865 👗 Top Class Call Girl Service Ban...
Call Girls Banaswadi Just Call 👗 7737669865 👗 Top Class Call Girl Service Ban...Call Girls Banaswadi Just Call 👗 7737669865 👗 Top Class Call Girl Service Ban...
Call Girls Banaswadi Just Call 👗 7737669865 👗 Top Class Call Girl Service Ban...amitlee9823
 
Top Rated Pune Call Girls Pashan ⟟ 6297143586 ⟟ Call Me For Genuine Sex Serv...
Top Rated  Pune Call Girls Pashan ⟟ 6297143586 ⟟ Call Me For Genuine Sex Serv...Top Rated  Pune Call Girls Pashan ⟟ 6297143586 ⟟ Call Me For Genuine Sex Serv...
Top Rated Pune Call Girls Pashan ⟟ 6297143586 ⟟ Call Me For Genuine Sex Serv...Call Girls in Nagpur High Profile
 
Stock Market Brief Deck (Under Pressure).pdf
Stock Market Brief Deck (Under Pressure).pdfStock Market Brief Deck (Under Pressure).pdf
Stock Market Brief Deck (Under Pressure).pdfMichael Silva
 
Booking open Available Pune Call Girls Talegaon Dabhade 6297143586 Call Hot ...
Booking open Available Pune Call Girls Talegaon Dabhade  6297143586 Call Hot ...Booking open Available Pune Call Girls Talegaon Dabhade  6297143586 Call Hot ...
Booking open Available Pune Call Girls Talegaon Dabhade 6297143586 Call Hot ...Call Girls in Nagpur High Profile
 
VIP Independent Call Girls in Mira Bhayandar 🌹 9920725232 ( Call Me ) Mumbai ...
VIP Independent Call Girls in Mira Bhayandar 🌹 9920725232 ( Call Me ) Mumbai ...VIP Independent Call Girls in Mira Bhayandar 🌹 9920725232 ( Call Me ) Mumbai ...
VIP Independent Call Girls in Mira Bhayandar 🌹 9920725232 ( Call Me ) Mumbai ...dipikadinghjn ( Why You Choose Us? ) Escorts
 
( Jasmin ) Top VIP Escorts Service Dindigul 💧 7737669865 💧 by Dindigul Call G...
( Jasmin ) Top VIP Escorts Service Dindigul 💧 7737669865 💧 by Dindigul Call G...( Jasmin ) Top VIP Escorts Service Dindigul 💧 7737669865 💧 by Dindigul Call G...
( Jasmin ) Top VIP Escorts Service Dindigul 💧 7737669865 💧 by Dindigul Call G...dipikadinghjn ( Why You Choose Us? ) Escorts
 

Recently uploaded (20)

Top Rated Pune Call Girls Aundh ⟟ 6297143586 ⟟ Call Me For Genuine Sex Servi...
Top Rated  Pune Call Girls Aundh ⟟ 6297143586 ⟟ Call Me For Genuine Sex Servi...Top Rated  Pune Call Girls Aundh ⟟ 6297143586 ⟟ Call Me For Genuine Sex Servi...
Top Rated Pune Call Girls Aundh ⟟ 6297143586 ⟟ Call Me For Genuine Sex Servi...
 
falcon-invoice-discounting-unlocking-prime-investment-opportunities
falcon-invoice-discounting-unlocking-prime-investment-opportunitiesfalcon-invoice-discounting-unlocking-prime-investment-opportunities
falcon-invoice-discounting-unlocking-prime-investment-opportunities
 
Top Rated Pune Call Girls Shikrapur ⟟ 6297143586 ⟟ Call Me For Genuine Sex S...
Top Rated  Pune Call Girls Shikrapur ⟟ 6297143586 ⟟ Call Me For Genuine Sex S...Top Rated  Pune Call Girls Shikrapur ⟟ 6297143586 ⟟ Call Me For Genuine Sex S...
Top Rated Pune Call Girls Shikrapur ⟟ 6297143586 ⟟ Call Me For Genuine Sex S...
 
Call Girls in New Friends Colony Delhi 💯 Call Us 🔝9205541914 🔝( Delhi) Escort...
Call Girls in New Friends Colony Delhi 💯 Call Us 🔝9205541914 🔝( Delhi) Escort...Call Girls in New Friends Colony Delhi 💯 Call Us 🔝9205541914 🔝( Delhi) Escort...
Call Girls in New Friends Colony Delhi 💯 Call Us 🔝9205541914 🔝( Delhi) Escort...
 
VIP Call Girl in Mumbai 💧 9920725232 ( Call Me ) Get A New Crush Everyday Wit...
VIP Call Girl in Mumbai 💧 9920725232 ( Call Me ) Get A New Crush Everyday Wit...VIP Call Girl in Mumbai 💧 9920725232 ( Call Me ) Get A New Crush Everyday Wit...
VIP Call Girl in Mumbai 💧 9920725232 ( Call Me ) Get A New Crush Everyday Wit...
 
Solution Manual for Principles of Corporate Finance 14th Edition by Richard B...
Solution Manual for Principles of Corporate Finance 14th Edition by Richard B...Solution Manual for Principles of Corporate Finance 14th Edition by Richard B...
Solution Manual for Principles of Corporate Finance 14th Edition by Richard B...
 
(INDIRA) Call Girl Mumbai Call Now 8250077686 Mumbai Escorts 24x7
(INDIRA) Call Girl Mumbai Call Now 8250077686 Mumbai Escorts 24x7(INDIRA) Call Girl Mumbai Call Now 8250077686 Mumbai Escorts 24x7
(INDIRA) Call Girl Mumbai Call Now 8250077686 Mumbai Escorts 24x7
 
From Luxury Escort Service Kamathipura : 9352852248 Make on-demand Arrangemen...
From Luxury Escort Service Kamathipura : 9352852248 Make on-demand Arrangemen...From Luxury Escort Service Kamathipura : 9352852248 Make on-demand Arrangemen...
From Luxury Escort Service Kamathipura : 9352852248 Make on-demand Arrangemen...
 
VIP Call Girl in Mumbai Central 💧 9920725232 ( Call Me ) Get A New Crush Ever...
VIP Call Girl in Mumbai Central 💧 9920725232 ( Call Me ) Get A New Crush Ever...VIP Call Girl in Mumbai Central 💧 9920725232 ( Call Me ) Get A New Crush Ever...
VIP Call Girl in Mumbai Central 💧 9920725232 ( Call Me ) Get A New Crush Ever...
 
Booking open Available Pune Call Girls Wadgaon Sheri 6297143586 Call Hot Ind...
Booking open Available Pune Call Girls Wadgaon Sheri  6297143586 Call Hot Ind...Booking open Available Pune Call Girls Wadgaon Sheri  6297143586 Call Hot Ind...
Booking open Available Pune Call Girls Wadgaon Sheri 6297143586 Call Hot Ind...
 
VIP Call Girl in Thane 💧 9920725232 ( Call Me ) Get A New Crush Everyday With...
VIP Call Girl in Thane 💧 9920725232 ( Call Me ) Get A New Crush Everyday With...VIP Call Girl in Thane 💧 9920725232 ( Call Me ) Get A New Crush Everyday With...
VIP Call Girl in Thane 💧 9920725232 ( Call Me ) Get A New Crush Everyday With...
 
VIP Independent Call Girls in Bandra West 🌹 9920725232 ( Call Me ) Mumbai Esc...
VIP Independent Call Girls in Bandra West 🌹 9920725232 ( Call Me ) Mumbai Esc...VIP Independent Call Girls in Bandra West 🌹 9920725232 ( Call Me ) Mumbai Esc...
VIP Independent Call Girls in Bandra West 🌹 9920725232 ( Call Me ) Mumbai Esc...
 
Kharghar Blowjob Housewife Call Girls NUmber-9833754194-CBD Belapur Internati...
Kharghar Blowjob Housewife Call Girls NUmber-9833754194-CBD Belapur Internati...Kharghar Blowjob Housewife Call Girls NUmber-9833754194-CBD Belapur Internati...
Kharghar Blowjob Housewife Call Girls NUmber-9833754194-CBD Belapur Internati...
 
VIP Independent Call Girls in Taloja 🌹 9920725232 ( Call Me ) Mumbai Escorts ...
VIP Independent Call Girls in Taloja 🌹 9920725232 ( Call Me ) Mumbai Escorts ...VIP Independent Call Girls in Taloja 🌹 9920725232 ( Call Me ) Mumbai Escorts ...
VIP Independent Call Girls in Taloja 🌹 9920725232 ( Call Me ) Mumbai Escorts ...
 
Call Girls Banaswadi Just Call 👗 7737669865 👗 Top Class Call Girl Service Ban...
Call Girls Banaswadi Just Call 👗 7737669865 👗 Top Class Call Girl Service Ban...Call Girls Banaswadi Just Call 👗 7737669865 👗 Top Class Call Girl Service Ban...
Call Girls Banaswadi Just Call 👗 7737669865 👗 Top Class Call Girl Service Ban...
 
Top Rated Pune Call Girls Pashan ⟟ 6297143586 ⟟ Call Me For Genuine Sex Serv...
Top Rated  Pune Call Girls Pashan ⟟ 6297143586 ⟟ Call Me For Genuine Sex Serv...Top Rated  Pune Call Girls Pashan ⟟ 6297143586 ⟟ Call Me For Genuine Sex Serv...
Top Rated Pune Call Girls Pashan ⟟ 6297143586 ⟟ Call Me For Genuine Sex Serv...
 
Stock Market Brief Deck (Under Pressure).pdf
Stock Market Brief Deck (Under Pressure).pdfStock Market Brief Deck (Under Pressure).pdf
Stock Market Brief Deck (Under Pressure).pdf
 
Booking open Available Pune Call Girls Talegaon Dabhade 6297143586 Call Hot ...
Booking open Available Pune Call Girls Talegaon Dabhade  6297143586 Call Hot ...Booking open Available Pune Call Girls Talegaon Dabhade  6297143586 Call Hot ...
Booking open Available Pune Call Girls Talegaon Dabhade 6297143586 Call Hot ...
 
VIP Independent Call Girls in Mira Bhayandar 🌹 9920725232 ( Call Me ) Mumbai ...
VIP Independent Call Girls in Mira Bhayandar 🌹 9920725232 ( Call Me ) Mumbai ...VIP Independent Call Girls in Mira Bhayandar 🌹 9920725232 ( Call Me ) Mumbai ...
VIP Independent Call Girls in Mira Bhayandar 🌹 9920725232 ( Call Me ) Mumbai ...
 
( Jasmin ) Top VIP Escorts Service Dindigul 💧 7737669865 💧 by Dindigul Call G...
( Jasmin ) Top VIP Escorts Service Dindigul 💧 7737669865 💧 by Dindigul Call G...( Jasmin ) Top VIP Escorts Service Dindigul 💧 7737669865 💧 by Dindigul Call G...
( Jasmin ) Top VIP Escorts Service Dindigul 💧 7737669865 💧 by Dindigul Call G...
 

ECONOMICS AND SOCIAL ISSUES PDF FOR RBI GRADE B EXAM

  • 1.
  • 2.
  • 3. The category of unemployment arising from the mismatch between the jobs available in the market and the skills of the available workers in the market is known as? A. Frictional Unemployment B. Cyclic Unemployment C. Structural Unemployment D. Disguised Unemployment E. None of the above Q.1
  • 4. Answer: Option C • Structural changes can be due to change in technology (from labour intensive technology to capital intensive technology) or change in the pattern of demand. • Thus, structural unemployment is a category of unemployment arising from the mismatch between the jobs available in the market and the skills of the available workers in the market.
  • 5. Labour Force Participation Rate (LFPR) is defined as the number of persons in the __________per 1000 persons. A. Employed persons B. Unemployed persons C. Total Population D. A & B E. None of the above Q.2
  • 7. Which one of the following is the correct sequence in the decreasing order of contribution of different sectors to the Gross Domestic Product of India? A. Services – Industry – Agriculture B. Services – Agriculture – Industry C. Industry – Services – Agriculture D. Industry – Agriculture – Services E. Agriculture – Industry – Services Q.3
  • 9. Research and Development based activity comes under ____________ activity and Retail trade falls under _________ activity. A. Primary, Quaternary B. Tertiary, Secondary C. Quaternary, Tertiary D. Quaternary, Secondary E. None of the above Q.4
  • 10. Answer: Option C • Quaternary activities are economic activities that centre around research and development. This primarily involves creating, organising and interpreting activities that focus on intellectual development. ➢ Quaternary activities typically centre on research and development and involve advanced services involving technical skills and specialised knowledge. Some examples of products and services offered by quaternary activities are robotics, the internet of things, 3D printing, quantum computing and nanotechnology. • Retail trade is a tertiary business activity that involves selling goods and services directly to customers and it occurs through local shops and retail stores devoted to selling customers. Other selling tactics of retail trading can include trucks, hand-to-hand cart selling, automatic vending machines, door-to-door selling, public distribution systems, street peddling and mail order. • Hence Option C is the answer.
  • 11. Which of the following phenomenon includes High inflation & high unemployment? A. Deflation B. Reflation C. Disinflation D. Skew inflation E. Stagflation Q.5
  • 12. Answer: Option E Reflation is a fiscal or monetary policy designed to expand output, stimulate spending, and curb the effects of deflation. Methods of reflation are reducing taxes, lowering interest rates, change in money supply, capital projects etc. Skewflation is a type of inflation in which the prices of a single commodity or a set of commodities rise while the overall price level remains stable Stagflation is a condition in which slow economic growth (stagnation), rising prices (inflation), and rising unemployment all happen at the same time. Although it is rare for slow economic growth and high inflation to coexist, it has happened in the past, and many believe it could happen again.
  • 13. Answer: Option E Deflation is the economic term used to describe the drop in prices for goods and services. Deflation slows down economic growth. It normally takes place during times of economic uncertainty when the demand for goods and services is lower, along with higher levels of unemployment. When prices fall, the inflation rate drops below 0%. Deflation (and inflation) rates can be calculated using the consumer price index (CPI). This index measures the changes in the price levels of a basket of goods and services. They can also be measured using the gross domestic product (GDP) deflator, which measures the price inflation Disinflation occurs when price inflation slows down temporarily. Unlike deflation, this is not harmful to the economy because the inflation rate is reduced marginally over a short-term period. Disinflation is the change in the rate of inflation. Prices do not drop during periods of disinflation and it does not signal an economic slowdown. Disinflation is demonstrated by a change in the inflation rate from one year to the next. So disinflation would be measured as a change of 4% from one year to 2.5% in the next.
  • 14. Which of the following statements is incorrect? A. The major reason for cyclical unemployment is a lack of demand in the economy and a slowdown in economic activity. B. Structural unemployment usually occurs due to the mismatch of skills. C. Disguised unemployment is when too many people are employed than what is required to produce efficiently. D. The minimum amount of unemployment that prevails in an economy due to workers quitting their previous jobs and are searching for new jobs is called Involuntary Unemployment E. None of the above Q.6
  • 15. Answer: Option D Cyclical unemployment is due to a deficiency or fall in effective demand from consumers which leads to a fall in production and low demand for labor. Structural unemployment refers to a situation that arises due to a change in the structure of the economy. Example: An economy transforms itself from a Labour-intensive economy to a Capital intensive economy. Disguised unemployment is when too many people are employed than what is required to produce efficiently. Involuntary unemployment refers to a situation where workers are seeking work and are willing to work but are unable to get work. Hence Option D is the answer. · The minimum amount of unemployment that prevails in an economy due to workers quitting their previous jobs and are searching for the new jobs is called Frictional Unemployment. · Voluntary unemployment refers to a situation where workers are either not seeking for work or are in transition from one job to another (quitting one job in search of another better job). · Seasonal unemployment occurs during certain seasons of the year. In some industries and occupations like agriculture, holiday resorts etc., production activities take place only in some seasons.
  • 16. What inflation rate RBI is mandated to maintain in the economy with a margin of 2% on either side? A. 4% B. 2% C. 6% D. 5% E. 8% Q.7
  • 17. Answer: Option A In May 2016, the Reserve Bank of India (RBI) Act, 1934 was amended to provide a statutory basis for the implementation of the flexible inflation targeting framework. The amended RBI Act also provides for the inflation target to be set by the Government of India, in consultation with the Reserve Bank, once in every five years. Accordingly, the Central Government notified in the Official Gazette 4 per cent Consumer Price Index (CPI) inflation as the target for the period from August 5, 2016 to March 31, 2021 with the upper tolerance limit of 6 per cent and the lower tolerance limit of 2 per cent. On March 31, 2021, the Central Government retained the inflation target and the tolerance band for the next 5-year period – April 1, 2021 to March 31, 2026.
  • 18. The National Statistical Organization (NSO) comes under which of the following Ministries of the Government of India? A. Ministry of Statistics and Programme Implementation B. Ministry of Finance C. Ministry of Commerce and Industry D. Ministry of Home Affairs E. Ministry of Labour and Employment Q.8
  • 19. Answer: Option A • The National Statistical Organization (NSO) is headed by a Director General and comes under the National Statistical Office of the Ministry of Statistics and Programme Implementation. It is responsible for conduct of large scale sample surveys in diverse fields on All India basis. • Primarily data are collected through nation-wide household surveys on various socio-economic subjects, Annual Survey of Industries (ASI), etc.
  • 20. Arrange the following committees in chronological order in terms of their formation: 1. Rangarajan Committee 2. Lakdawala Committee 3. Alagh Committee 4. Tendulkar Committee Which of the statements given above is/are correct? A. 3-2-4-1 B. 1-4-2-3 C. 1-3-2-4 D. 4-2-1-3 E. 2-1-3-4 Q.9
  • 22. Which of the following is incorrect regarding Wholesale Price Index? A. It is published by the Office of Economic Adviser, Ministry of Commerce and Industry B. The base year of it is 2011-12 C. It includes goods only. D. The highest weightage in the index is given to the primary articles. E. None of the above Q.10
  • 23. Answer: Option D Wholesale Price Index (WPI) Wholesale Price Index (WPI) measures the average change in the prices of commodities for bulk sale at the level of early stage of transactions. It is published by the Office of Economic Adviser, Ministry of Commerce and Industry The base year of it is 2011-12 The index basket of the WPI covers commodities falling under the three Major Groups namely Primary Articles, Fuel and Power and Manufactured products.
  • 25. Montreal Protocol aims to achieve which of the following? A. Protect human health and the environment from anthropogenic emissions and releases of mercury B. Protect the ozone layer by phasing out the production of numerous substances that are responsible for ozone depletion C. Limit Global warming to the 2 degree Celsius increase from pre industrial level D. Provide access to Genetic Resources and the Fair and Equitable Sharing of Benefits Arising from their Utilization E. None of the above Q.11
  • 26. Answer: Option B The Montreal Protocol on Substances that Deplete the Ozone Layer is an international treaty designed to protect the ozone layer by phasing out the production of numerous substances that are responsible for ozone depletion.
  • 27. Which of the following is not a component of India's Forex reserve? A. Foreign Currency assets B. Gold reserves C. Special Drawing Rights (SDRs) D. India's reserve position in the IMF E. Foreign Assets with individuals Q.12
  • 28. Answer: Option E • Foreign exchange assets serve many purposes but are most significantly held to ensure that the central bank has backup funds if the national currency rapidly devalues or becomes altogether insolvent. • The four components of forex reserves are foreign currency assets, gold, special drawing rights and the reserve position in the International Monetary Fund.
  • 29. Which kind of policy involves raising taxes or cutting government spending, so that it cuts up on the aggregate demand (thus, economic growth) to reduce the inflationary pressures in the economy? A. Neutral Fiscal policy B. Contractionary Fiscal policy C. Expansionary Fiscal Policy D. Expansionary Monetary Policy E. Contractionary Monetary Policy Q.13
  • 31. There has been persistent deficit budget year after year. Which action/actions of the following can be taken by the Government to reduce the deficit? 1. Reducing revenue expenditure 2. Introducing new Welfare schemes 3. Rationalizing subsidies 4. Reducing import duty Select the correct answer using the code given below: A. 1 and 3 only B. 2 and 3 only C. 3 and 4 only D. 1 only E. 1, 2, 3 and 4 Q.14
  • 32. Answer: Option A • Budget deficit is used to define a status of financial health in which expenditures exceed revenue. • Introducing new welfare scheme by budgetary support will expand the budget expenditure which will widen the deficit. • Similarly, reducing import duty will lead to less revenue for the government, which leads to increase in budget deficit. • Rationalizing subsidies and reducing revenue expenditure are two direct ways of reducing the fiscal burden of the government of India
  • 33. The term National Income represents____________ A. Gross national product at market prices minus depreciation B. Gross national product at market prices minus depreciation plus net factor income from abroad C. Gross national product at market prices minus depreciation and indirect taxes plus subsidies D. Gross national product at market prices minus net factor income from abroad E. None of the above Q.15
  • 34. Answer: Option C NNP at FC = gross national product at market prices - depreciation - indirect taxes + subsidies
  • 35. There will be _________ Budget, if the sum of capital receipts and revenue receipts is greater than the sum of Capital Expenditure and Revenue Expenditure. A. Surplus Budget B. Balanced Budget C. Revenue Deficit in the Budget D. Deficit Budget E. None of the above Q.16
  • 37. Which of the following best describes the meaning of Fiscal Deficit? A. The excess of revenue expenditure over revenue receipts B. The excess of capital expenditure over capital receipts C. The excess of total expenditure over total receipts excluding borrowings D. The excess of total expenditure over total receipts E. None of the above Q.17
  • 38. Answer: Option C Fiscal Deficit - The difference between total revenue and total expenditure of the government is termed as fiscal deficit. It is an indication of the total borrowings needed by the government. While calculating the total revenue, borrowings are not included.
  • 39. The Sustainable Development Goals or Global Goals are a collection of 17 interlinked global goals designed to be a “blueprint to achieve a better and more sustainable future for all". These goals are intended to be achieved by which year? A. 2025 B. 2027 C. 2030 D. 2035 E. 2040 Q.18
  • 40. Answer: Option C • The United Nations Sustainable Development Goals (SDGs) are targets for global development adopted in September 2015, set to be achieved by 2030. All countries of the world have agreed to work towards achieving these goals. • Our SDG Tracker presents data across all available indicators from the Our World in Data database, using official statistics from the UN and other international organizations. It is a free, open-access publication that tracks global progress towards the SDGs and allows people around the world to hold their governments accountable to achieving the agreed goals. • The 17 Sustainable Development Goals are defined in a list of 169 SDG Targets. Progress towards these Targets is agreed to be tracked by 232 unique Indicators. • 2030 Agenda for Sustainable Development Goals which has been agreed upon by all 193 member states was organised by United Nations General Assembly
  • 41. As part of 2030 Agenda for sustainable development, what is the total number of targets to be achieved under the 17 SDGs? A. 189 B. 180 C. 179 D. 169 E. 164 Q.19
  • 42. Answer: Option D • The United Nations Sustainable Development Goals (SDGs) are targets for global development adopted in September 2015, set to be achieved by 2030. All countries of the world have agreed to work towards achieving these goals. • Our SDG Tracker presents data across all available indicators from the Our World in Data database, using official statistics from the UN and other international organizations. It is a free, open-access publication that tracks global progress towards the SDGs and allows people around the world to hold their governments accountable to achieving the agreed goals. • The 17 Sustainable Development Goals are defined in a list of 169 SDG Targets. Progress towards these Targets is agreed to be tracked by 232 unique Indicators.
  • 43. Consider the following statements regarding Human Development Index (HDI): 1. The Human Development Index (HDI) is a composite index that measures the average achievements in a country in three basic dimensions of human development. 2. The basic dimensions are a long and healthy life, knowledge and a decent standard of living. 3. World Bank releases the index report. Which of the following statement(s) is/are correct? A. Only 1 & 2 B. Only 2 &3 C. 1, 2 and 3 D. Only 1 E. Only 2 Q.20
  • 44. Answer: Option A • The Human Development Index (HDI) is a summary measure of average achievement in key dimensions of human development: a long and healthy life, being knowledgeable and having a decent standard of living. • The Human Development Index (HDI) is a composite index that measures the average achievements in a country in three basic dimensions of human development. • These basic dimensions are a long and healthy life, knowledge and a decent standard of living.
  • 45. Consider the following pairs regarding different sectors of the economy: 1. Primary Sector: It contains all of the economic activities under which the raw materials extracted are processed and manufactured into final goods. 2. Secondary Sector: The economic activities which take place while exploiting the natural resources fall under it. 3. Quinary Sector: The highest level of decision-makers in governments and the private corporate sector fall under it. 4. Quaternary Sector: It is known also as ‘knowledge’ sector, the activities related to education, research and development, etc. come under it. Which of the following pairs are correctly matched? A. 3 only B. 2 only C. 1 and 2 only D. 1, 2 and 3 only E. 3 & 4 only Q.21
  • 46. Answer: Option E • Primary Sector: The economic activities which take place while exploiting the natural resources fall under it, such as mining, agricultural activities, oil exploration, etc. When the agriculture sector (one of the sub-sectors of the primary sector) contributes a minimum of half of the national income and livelihood in a country it is called an agrarian economy. Hence statement 1 is incorrect. • Secondary Sector: It contains all of the economic activities under which the raw materials extracted out of the primary sector are processed (also called the industrial sector). One of its sub-sectors, manufacturing, has proved to be the largest employer across the western developed Economies. Hence statement 2 is incorrect. • Tertiary Sector: All of the economic activities where services are produced falls in this sector, such as education, healthcare, banking, communication, etc. When this sector contributes minimum half of the national income and livelihood in a country it is called a service economy. Later on, experts created two more sectors of economy— quaternary and quinary. Though, they are subsectors of the tertiary sector. • Quaternary Sector: Known also as ‘knowledge’ sector, the activities related to education, research and development, etc. come under it. The sector plays the most important role in defining the quality of the human resources an economy has. Hence statement 4 is correct. • Quinary Sector: All activities where top decisions are made fall under it. The highest level of decision makers in governments (inclusive of their bureaucracy) and the private corporate sector fall under it. The number of people involved in this sector is very low rather they are considered the ‘brain’ behind socio-economic performance of an economy. Hence statement 3 is correct. Therefore, the answer is Option E
  • 47. Which of the following services is not a part of services sector in India? A. Transport and Communication B. Financial Services C. Defence Services D. Utility Services E. Public Administration Q.22
  • 48. Answer: Option D •According to Economic survey, The classification is mentioned in the below table.
  • 49. "Absence of minimum income to get the minimum needs of life" is concerned with which of the following types of poverty? A. Absolute poverty B. Relative poverty C. Situational poverty D. Intergenerational poverty E. Multidimensional poverty Q.23
  • 50. Answer: Option A Absolute poverty can be defined as the state in which a subject lacks the means to meet his or her basic needs. Such basic needs are often listed in international poverty reduction programs, and usually include food, water, shelter, basic education, and basic medical care. Hence option A is the answer. Extreme poverty is typically defined as a state in which a person lacks access to all, or several, of the goods needed for meeting these basic needs. Relative poverty is closely associated with the issues of inequality. It occurs when people in a country do not enjoy a certain minimum level of living standards as compared to the rest of the population and so would vary from country to country, sometimes within the same country. Multidimensional poverty is made up of several factors that constitute poor people’s experience of deprivation – such as poor health, lack of education, inadequate living standard, lack of income (as one of several factors considered), disempowerment, poor quality of work and threat from violence.
  • 51. Mahalanobis Plan Model adopted in India in the Mid-fifties aimed at? A. Building a strong defence industry base B. Setting up heavy industries which were capital intensive C. Curbing inflation in the economy D. Removing unemployment within a short period E. None of the above Q.24
  • 52. Answer: Option B • The correct answer is Setting up heavy industries which were capital intensive. • Mahalanobis Plan Model adopted in India in the Mid-fifties aimed at Setting up heavy industries which were capital intensive. ➢ In the 1950s, an Old Russian Model was Indianized by P.C. Mahalanobis. ➢ This model is known to have set the statistical foundations for state-directed investments and created the intellectual underpinnings of the license-raj through an elaborate input-output model. ➢ This Model suggested that there should be an emphasis on the heavy industries, which can lead the Indian Economy to a long-term higher growth path. ➢ India's second five-year plan and Industrial policy Resolution 1956, which paved the way for the development of the Public Sector and license raj. ➢ He was the founder of the Indian Statistical Institute and a close aide of Pandit Jawahar Lal Nehru.
  • 53. Which of the following statements is not correct? A. The first five year plan was launched from 1st April, 1951 B. The first five year plan gave emphasis on Heavy Industrialization C. The first five year plan was from 1951-1956 D. The first five year plan was based on the Harrod- Domar model E. None of the above Q.25
  • 54. Answer: Option B • The Second Five year plan emphasized on Heavy industrialization which was based on P.C. Mahalonobis.
  • 55.
  • 56. Which of the following is not the correct match? A. Fiscal Deficit – Excess of Total Expenditure over total receipts less borrowings B. Budget Deficit – Excess of Total Expenditure over Total Receipts C. Revenue Deficit – Excess of Total Expenditure over Total revenue receipts D. Primary Deficit – Excess of total Expenditure over total receipts less borrowings and interest payments E. None of the Above Q.26
  • 58. ________ is the change in costs of goods and services but does not include those from the food and energy sectors. A. Retail Inflation B. Hyper Inflation C. Core Inflation D. Stagflation E. None of the above Q.27
  • 59. Answer: Option C Core inflation is the change in costs of goods and services, but does not include those from the food and energy sectors. This measure of inflation excludes these items because their prices are much more volatile. Headline inflation is a measure of the total inflation within an economy, including commodities such as food and energy prices (e.g., oil and gas), which tend to be much more volatile and prone to inflationary spikes
  • 60. In India, CPI (Rural/Urban/Combined) is published by the _______ and CPI (IW/AL) is published by Labour Bureau in the Ministry of Labour and Employment. A. Ministry of Statistics and Programme Implementation B. Ministry of Corporate Affairs C. Ministry of Finance D. Ministry of Home Affairs E. Reserve Bank of India (RBI) Q.28
  • 61. Consumer Price Index • It measures price changes from the perspective of a retail buyer. It is released by the National Statistical Office (NSO). • The CPI calculates the difference in the price of commodities and services such as food, medical care, education, electronics etc, which Indian consumers buy for use. • The CPI has several sub-groups including food and beverages, fuel and light, housing and clothing, bedding and footwear. • Four types of CPI are as follows: ➢ CPI for Industrial Workers (IW) ➢ CPI for Agricultural Labourer (AL) ➢ CPI for Rural Labourer (RL) ➢ CPI (Rural/Urban/Combined) Of these, the first three are compiled by the Labour Bureau in the Ministry of Labour and Employment. Fourth is compiled by the NSO in the Ministry of Statistics and Programme Implementation. • Base Year for CPI is 2012 Answer: Option A
  • 62. Which type of fiscal policy is being described in the following lines? This policy is designed to boost the economy. It is mostly used in times of high unemployment and recession. It leads to the government lowering taxes and spending more, or one of the two. The aim is to stimulate the economy and ensure consumers' purchasing power does not weaken. A. Neutral Fiscal policy B. Contractionary Fiscal policy C. Expansionary Fiscal Policy D. Information inadequate E. None of the Aboveand Q.38
  • 63. Expansionary fiscal policy: This policy is designed to boost the economy. It is mostly used in times of high unemployment and recession. It leads to the government lowering taxes and spending more or one of the two. The aim is to stimulate the economy and ensure consumers' purchasing power does not weaken. Hence Option C is the correct answer. There are two key tools of the fiscal policy: •Taxation: Funds in the form of direct and indirect taxes, capital gains from investment, etc, help the government function. Taxes affect the consumer's income and changes in consumption lead to changes in real gross domestic product (GDP). •Government spending: It includes welfare programmes, government salaries, subsidies, infrastructure, etc. Government spending has the power to raise or lower real GDP, hence it is included as a fiscal policy tool. Answer: Option C
  • 64. The Dependency Ratio in India is declining because A. Population of 0-14 years is relatively high B. Population of 64 years and above is relatively high C. Population of 15-64 years is relatively high D. The population of 0-14 years and 64 years and above together are relatively high E. Population of 15-64 is relatively low. Q.39
  • 65. Key Points •The Dependency Ratio calculates the number of young and old people and divides them by the total population including working adults. •When the dependency ratio is high, it suggests that there is a large elderly/youthful population. •The higher the dependency ratio, the more the working-age population has to pay to sustain its dependents – unless services or social payments are reduced. •When the dependency ratio is low and working age population is high, it suggests that there is a working age population. Hence option C is the answer. Age dependency ratio is the ratio of dependents--people younger than 15 or older than 64-- to the working-age population--those ages 15-64. Answer: Option C
  • 66. Which of the following is/are correct about gender budgeting? 1. It is a tool for gender mainstreaming. 2. Gender budgeting seeks to create a separate budget. 3. Gender budgeting recognizes that if gender disparities are to be addressed, adequate monetary allocations are to be made for the achievement of desired outcomes Select the correct answer using the code given below: A. 1 and 2 only B. 2 and 3 only C. 1 and 3 only D. 1, 2 and 3 only E. 1 only Q.40
  • 67. Gender Budgeting (GB) 1. GB is concerned with gender sensitive formulation of legislation, programmes and schemes; allocation of resources; implementation and execution; audit and impact assessment of programmes and schemes; and follow-up corrective action to address gender disparities. 2. A powerful tool for achieving gender mainstreaming so as to ensure that benefits of development reach women as much as men. 3. Does not seek to create a separate budget but seeks affirmative action to address specific needs of women. 4. Monitors expenditure and public service delivery from a gender perspective. 5. Entails dissection of the Government budgets to establish its gender differential impacts and to ensure that gender commitments are translated in to budgetary commitments. 6. It is a tool for gender mainstreaming. 7. Gender budgeting uses the budget as an entry point to apply a gender lens to the entire policy process. 8. Gender budgeting also recognizes that if gender disparities are to be addressed, adequate monetary allocations are to be made for the achievement of desired outcomes Answer: Option C
  • 68. Which of the following is/are the part of the Liquid Adjustment Facility (LAF)? 1.Repo Rate 2.Marginal Standing Facility 3.Bank Rate 4.Standing Deposit Facility Choose the correct answer using the codes given below: A. 1 only B. 2 & 3 C. 1, 2 and 4 D. 1, 2 and 3 E. None of the above Q.41
  • 70. The Global Financial Stability Report provides an assessment of the global financial system and markets, and addresses emerging market financing in a global context. It focuses on current market conditions, highlighting systemic issues that could pose a risk to financial stability and sustained market access by emerging market borrowers. The report is published by__________________. A. International Monetary Fund B. United Nations Conference on Trade and Development C. World Economic Forum D. World Bank E. UNESCO Q.42
  • 71. • The Global Financial Stability Report provides an assessment of the global financial system and markets, and addresses emerging market financing in a global context. It focuses on current market conditions, highlighting systemic issues that could pose a risk to financial stability and sustained market access by emerging market borrowers. The Report draws out the financial ramifications of economic imbalances highlighted by the IMF's World Economic Outlook. It contains, as special features, analytical chapters or essays on structural or systemic issues relevant to international financial stability. • The Global Financial Stability Report (GFSR) is a survey by the IMF staff published twice a year, in the spring and fall. • The World Economic Outlook (WEO) and the Global Financial Stability Report (GFSR) are the reports published by the IMF. • Global Gender Gap Report, Global Risk Report, and the Travel and Tourism Competitiveness Report are the reports published by WEF. • The World Investment Report is released by the United Nations Conference on Trade and Development. • Global Economic Prospects, Ease of Doing Business, World Development Report are released by the World Bank. Answer: Option A
  • 72. “Fiscal Stimulus” is provided to different sectors of an economy to promote the growth. Which of the following measure does not constitute fiscal stimulus? A. Input Subsidies B. Monetary incentives C. Export subsidies D. Increasing Taxes E. Schemes for low interest rate Q.43
  • 73. A stimulus package is a number of incentives and tax rebates offered by a government to boost spending in a bid to pull a country out of a recession or to prevent an economic slowdown. • A stimulus package can either be in the form of a monetary stimulus or a fiscal stimulus. • A monetary stimulus involves cutting interest rates to stimulate the economy. • When interest rates are cut, there is more incentive for people to borrow as the cost of borrowing is reduced. • An increase in borrowing means there’ll be more money in circulation, less incentive to save, and more incentive to spend. • Lowering interest rates could also weaken the exchange rate of a country, thereby leading to a boost in exports. • When exports are increased, more money enters the economy, encouraging spending and stirring up the economy Answer: Option D
  • 74. Demographic Transition is described as A. a stage in which the population growth of the country is zero B. a process where there is a decline in the birth-rate C. A long-term trend of declining birth and death rates D. a process where there is a decline in the death rate E. a process where there is stagnancy in the society's population Q.44
  • 75. •Demographic Transition is a process of change in a society's population overtime. It is a long-term trend of declining birth and death rates, resulting in substantive change in the age distribution of a population. Hence option C is the answer. •It can be used to describe and predict the future population of any area. •It shows changes in birth rate and death rate and consequently on the growth rate of population. •It is the relationship between economic development and population growth. •Demographer Warren Thompson first introduced the demographic transition model in 1929. Answer: Option C
  • 76. With reference to Gross Value Added (GVA), which of the following statements is/are correct? A. It is defined as the value of output minus the value of intermediate consumption. B. Its estimate is released by National Statistical Office. C. From an economics perspective, it represents the supply side. D. In India, GVA is calculated at basic prices. E. All of the above Q.45
  • 77. Answer: Option E Gross Value Added • In 2015, India opted to make major changes to its compilation of national accounts and decided to bring the whole process into conformity with the United Nations System of National Accounts (SNA) of 2008. • The SNA is the internationally agreed standard set of recommendations on how to compile measures of economic activity. • It describes a coherent, consistent and integrated set of macroeconomic accounts in the context of a set of internationally agreed concepts, definitions, classifications and accounting rules. • As per the SNA, GVA is defined as the value of output minus the value of intermediate consumption and is a measure of the contribution to growth made by an individual producer, industry or sector. • It provides the rupee value for the number of goods and services produced in an economy after deducting the cost of inputs and raw materials that have gone into the production of those goods and services. • It can be described as the main entry on the income side of the nation’s accounting balance sheet, and from an economics perspective represents the supply side. • Earlier, India had been measuring GVA at ‘factor cost’ till the new methodology was adopted in which GVA at ‘basic prices’ became the primary measure of economic output. • GVA at basic prices will include production taxes and exclude production subsidies. • GVA at factor cost included no taxes and excluded no subsidies. • The base year has also been shifted to 2011-12 from the earlier 2004-05.
  • 78. Which of the following term denotes the average income earned by a person in India? A. National Income B. Per capita income C. Personal income D. Personal Disposable Income E. None of the above Q.46
  • 79. Answer: Option B • Per capita income (PCI) or average income measures the average income earned per person in a given area (city, region, country, etc.) in a specified year. It is calculated by dividing the area's total income by its total population. Per capita income is national income divided by population size.
  • 80. Why is India regarded as a country with a "Demographic Dividend”? A. Its high population in the age group below 15 years. B. Its high population in the age group of 15-64 years. C. Its high population in the age group above 65 years. D. Its high total population. E. Its high population in the age group of 20-80 years. Q.47
  • 81. Answer: Option B The demographic dividend is the economic growth potential that can result from shifts in a population's age structure, mainly when the share of the working-age population (15 to 64) is larger than the non-working-age share of the population (14 and younger, and 65 and older). Demographic dividend in India : • As populations in countries such as China, US, and Japan is getting older, India's population is getting younger. • India's working-age population is now increasing because of rapidly declining birth and death rates. • India's age dependency ratio, the ratio of dependents (children and the elderly) to the working-age population (14- to 65-year-olds), is expected to only start rising in 2040, as per UN estimates. This presents a golden opportunity for economic growth. • However, this growth will depend on those in the working-age population actually working. • India's labour force participation rate is declining, especially among rural youth (15- to 29-year-olds) and women. • For India to harness the power of its favourable demographics, India's labour force needs to be empowered with the right skills for the modern economy. Hence Option B is the answer.
  • 82. The literacy rate in India is measured after which age? A. 5 years and above B. 7 years and above C. 9 years and above D. 11 years and above E. 13 years and above Q.48
  • 83. Answer: Option B The literacy rate is the total percentage of the population of an area at a particular time aged seven years or above who can read and write with understanding. Hence Option B is the answer.
  • 84. Consider the following statements with respect to World Trade Organisation (WTO): 1. To facilitate the implementation, administration, and operation of a trade agreement 2. To carry out periodic reviews of the trade policies of its member countries 3. To promote international monetary cooperation Which of the above statement/s is/are correct? A. 1 and 2 B. 2 only C. 3 only D. 1 and 3 E. 2 and 3 Q.49
  • 85. Answer: Option A Functions of the World Trade Organisation: At the heart of the Organisation are the WTO agreements, negotiated and signed by the bulk of the world's trading nations. The goal is to help producers of goods and services, exporters, and importers conduct their business. The WTO's overriding objective is to help trade flow smoothly, freely, fairly, and predictably. • It shall facilitate the implementation, administration, and operation of the WTO trade agreements, such as multilateral trade agreements, plurilateral trade agreements. • It shall monitor national trade policies. The WTO was founded on certain guiding principles—non-discrimination, free trade, open, fair and undistorted competition, etc. In addition, it has special concern for developing countries. It is the function of International Monetary Fund to promote international monetary cooperation. Therefore, Option A is the correct answer.
  • 86. Which of the following institutions is not part of the World Bank community? A. International Bank for Reconstruction and Development (IBRD) B. International Development Agency (IDA) C. Bank for International Settlements (BIS) D. Multilateral Investment Guarantee Agency (MIGA) E. All of the above are a part of World Bank Community Q.50
  • 87. Answer: Option C The World Bank Group comprises five constituent institutions: the International Bank for Reconstruction and Development (IBRD), the International Development Association (IDA), the International Finance Corporation (IFC), the Multilateral Investment Guarantee Agency (MIGA), and the International Centre for Settlement of Investment Disputes (ICSID). Established in 1930, the BIS is owned by 63 central banks, representing countries from around the world that together account for about 95% of world GDP. Its head office is in Basel, Switzerland and it has two representative offices: in Hong Kong SAR and in Mexico City, as well as Innovation Hub Centres around the world. The BIS's mission is to support central banks' pursuit of monetary and financial stability through international cooperation, and to act as a bank for central banks.
  • 88. With reference to ‘G20’, which of the following statements is/are correct? 1.The G20 comprises 19 countries and the European Union. 2.The G20 was founded in 1999 after the Asian financial crisis and later was designated the “premier forum for international economic cooperation”. 3.Since 2011, the G20 Summit is held annually, under the leadership of a rotating Presidency and latest G20 Summit is held in Saudi Arabia under its presidency. Select the correct statement/s: A. Only 1 B. Only 2 C. Only 2 & 3 D. Only 1 & 2 E. Only 1 & 3 Q.51
  • 89. Answer: Option B •Introduction The Group of G20 (G20) comprises Argentina, Australia, Brazil, Canada, China, France, Germany, India, Indonesia, Italy, Japan, Republic of Korea, Mexico, Russia, Saudi Arabia, South Africa, Turkey, United Kingdom, and the United States along with the European Union and African Union. •G20 members currently account for more than 80% of world GDP, 75% of global trade, and 60% of the global population. Origin and Evolution •The G20 was founded in 1999 after the Asian financial crisis as a forum for the Finance Ministers and Central Bank Governors to discuss global economic and financial issues. T •he G20 was later upgraded to the level of Heads of State/Government and was designated the “premier forum for international economic cooperation”. •Since 2011, the G20 Summit is held annually, under the leadership of a rotating Presidency. The G20 initially focused largely on broad macroeconomic policy, but it has since expanded its ambit to include trade, climate change, sustainable development, energy, environment, climate change, anti-corruption etc. •India holds the Presidency of the G20 from 1 December 2022 to 30 November 2023. Hence statement 4 is incorrect.
  • 90. Which of the following is not the function of the International Monetary Fund? A. The IMF manages the international monetary system for global payments. B. IMF advises on macroeconomic policies and exchange rates. C. It assesses and regulates political sectors in member countries. D. The IMF offers financial assistance for balance of payments discrepancies. E. Its role includes promoting economic growth and employment within nations. Q.52
  • 91. Answer: Option C Functions of IMF 1. The International Monetary Fund (IMF) plays a vital role in creating and maintaining the international monetary system, which facilitates international payments between countries. 2. The IMF focuses on advising and monitoring the macroeconomic policies of member countries, particularly those affecting exchange rates, government budgets, and money and credit management. 3. As part of its responsibilities, the IMF assesses a country's financial sector and regulatory policies, as well as structural policies in the macro economy that impact the labor market and employment. 4. In addition to its advisory role, the IMF is authorized to provide financial assistance to nations facing balance of payments discrepancies. 5. It contributes to economic stability and growth while striving to maintain high levels of employment within member countries. Hence option C is the correct answer.
  • 92. Poverty Gap is defined as _________ A. the ratio by which the mean income of the poor falls below the poverty line. B. Gap between rich and poor. C. Gap between developed nation and developing nation. D. the intensity of poverty in relative terms of different sections of a country. E. None of the above Q.53
  • 93. Answer: Option A 1. According to the World Bank, poverty gap is the mean shortfall from the poverty line (counting the non-poor as having zero shortfall), expressed as a percentage of the poverty line. Poverty gap measures the intensity of poverty. It shows the extent to which individuals on average fall below the poverty line. 2. The poverty gap is a ratio showing the average shortfall of the total population from the poverty line—the minimum level of income required to secure the basic necessities for survival. In other words, it reflects the intensity of poverty in a nation. 3. The poverty gap reflects the intensity of poverty in a nation, showing the average shortfall of the total population from the poverty line. 4. The poverty gap is an indicator produced by the World Bank, which measures poverty by looking at per capita income and consumption in households. 5. The data is available for 115 countries and is updated semi-annually in April and September. 6. The poverty gap statistic is most valuable to economists and government officials for calculating the poverty gap index.
  • 94. NABARD was established on the recommendation of which committee? A. B. Sivaramman committee B. Narasimhan committee C. Kotak committee D. Naresh Chandra committee E. Narayan Murthy committee Q.54
  • 95. Answer: Option A The recognition of the importance of institutional credit in boosting rural economy by the Government of India led to the inception of a Committee to review the Arrangements for Institutional Credit for Agriculture and Rural Development (CRAFICARD). This was established under the able Chairmanship of Shri B. Sivaraman, former Member of Planning Commission on 30 March 1979. Based on the Committee’s interim report recommendation, the creation of National Bank for Agriculture and Rural Development (NABARD) was approved by the Parliament through Act 61 of 1981. ✓ Established: NABARD was established on 12th July 1982 on the recommendation of Committee to Review the Arrangements For Institutional Credit for Agriculture and Rural Development (CRAFICARD) which is also known as the Sivaraman Comittee. ✓ NABARD is the apex organisation related to financing in the agricultural sector. ✓ Headquarter: Mumbai, Maharashtra.
  • 96. Right to Education 2009 Act provides free and compulsory education to all children of the age of ________? A. 5-15 years B. 6-14 years C. 7-15 years D. 5-14 years E. 6-15 years Q.55
  • 97. Answer: Option B The Right of Children to Free and Compulsory Education Act, 2009 (the Act) provides for free and compulsory education to all children of the age of six to fourteen years.
  • 98. Which of the following is the second stage in the Theory of Demographic Transition? [A] High and fluctuating birth and death rates will almost neutralize each other [B] Death rate is decreasing while the birth rate remains constant at a high level [C] Birth rate as compared to the death rate declines more rapidly. As a result, the population grows at a diminishing rate [D] Birth rate is approximately equal to the death rate and there is little population growth. [E] None of the above Q.56
  • 100. As per SECC-2011, what percentage of total households live in rural India? [A] 52.37% [B] 67.37% [C] 79.37% [D] 63.37% [E] 73.37% Q.57
  • 101. Answer: Option E As per SECC-2011, out of the 24.49 crore households in India, 17.97 crore households is in villages which is 73.37% of all households in India.
  • 102. Which amendment act added the part IX to the Constitution of India named ‘Panchayati Raj’? [A] 76th Amendment Act [B] 75th Amendment Act [C] 73rd Amendment Act [D] 74th Amendment Act [E] None of the above Q.58
  • 103. Answer: Option C The Constitution (Seventy- Third Amendment) Act, 1992, which came into force w.e.f. 24th April, 1993, inserted Part IX in the Constitution of India and accorded Panchayats a Constitutional status as institutions of local self-governance for rural India.
  • 104. Neonatal mortality Rate (NMR) is the number of neonatal deaths per 1000 live births. A neonatal death is defined as a death during which of the following periods of life? [A] First 28 days of life [B] First 31 days of life [C] First 365 days of life [D] First 12 days of life [E] First 7 days of life Q.59
  • 105. Answer: Option A Neonatal death is defined as the death of a live born infant, regardless of gestational age at birth, within the first 28 completed days of life. The Maternal Mortality Ratio (MMR) is defined as the number of maternal deaths during a given time period per 100,000 live births.
  • 106. The factors (for example, adverse economic conditions caused by poverty, low productivity, unemployment, exhaustion of natural resources and natural calamities) that compel or force a person, due to various reasons, to leave that place and go to some other place are known as? [A] Pull factors [B] Push factors [C] Push back factors [D] Social factors [E] Political factors Q.60
  • 107. Answer: Option B Explanation: Push Factors The push factors are those that compel or force a person, due to various reasons, to leave that place and go to some other place. For example, adverse economic conditions caused by poverty, low productivity, unemployment, exhaustion of natural resources and natural calamities may compel people to leave their native place in search of better economic opportunities.
  • 108. Which of the following committee recommended the establishment of Regional Rural Banks? [A] Khusro Committee [B] Narsimham Committee [C] Dutt Committee [D] C Rangarajan Committee [E] Kotak Committee Q.61
  • 109. Answer: Option B RRBs were set up as government-sponsored, regional based rural lending institutions under the Regional Rural Banks Act, 1976. The RRBs were established as per the recommendations of the Narasimham Committee to cater to the rural credit needs of the farming and other rural communities.The Prathama Grameen Bank was the first bank to be established on 02nd October 1975. The Syndicate Bank became the first commercial bank to sponsor the Prathama Grameen Bank RRB
  • 110. Lending to which of the following sectors is not a part of priority sector lending? A. Micro, Small and Medium Enterprises B. Housing for poor C. Export credit D. Iron and Steel industry E. Social Infrastructure Q.62
  • 112. It is an international agreement linked to the United Nations Framework Convention on Climate Change, which commits its Parties by setting internationally binding emission reduction targets. Recognizing that developed countries are principally responsible for the current high levels of GHG emissions in the atmosphere as a result of more than 150 years of industrial activity, the Protocol places a heavier burden on developed nations under the principle of "common but differentiated responsibilities. Which of the following protocol is mentioned in the passage given above? A. Montreal Protocol B. Kyoto Protocol C. Cartagena Protocol D. Nagoya Protocol E. None of the above Q.63
  • 113. Kyoto Protocol is an international agreement linked to the United Nations Framework Convention on Climate Change, which commits its Parties by setting internationally binding emission reduction targets. Recognizing that developed countries are principally responsible for the current high levels of GHG emissions in the atmosphere as a result of more than 150 years of industrial activity, the Protocol places a heavier burden on developed nations under the principle of "common but differentiated responsibilities. “The Protocol was adopted in Japan, on 11 December 1997 and entered into force on 16 February 2005. The detailed rules for the implementation of the Protocol were adopted at COP 7 in Marrakesh, Morocco, in 2001, and are referred to as the "Marrakesh Accords." It covered few market based mechanisms (Flexible Market Mechanisms) to reduce the emissions. •Most nations have ratified the treaty. The USA is a notable exception to this. It takes the stand that having binding targets only for developed countries and not polluting countries like China and India is potentially harmful for its own economy. Canada withdrew from the Kyoto Protocol in 2012. •The targets are for the following greenhouse gases/gas groups: carbon dioxide, nitrous oxide, methane, sulphur hexafluoride, hydrofluorocarbons and perfluorocarbons. •The first commitment period for the agreement was from 2008 to 2012. •Apart from national measures, the agreement has three mechanisms that are means to achieve the Kyoto targets: • International Emissions Trading • Clean Development Mechanism • Joint Implementation Answer: Option B
  • 114. Clean Development Mechanism and Joint implementation are associated with: A. Kyoto Protocol B. Cartegena Protocol C. Nagoya Protocol D. Montreal Protocol E. None of the Above Q.64
  • 116. Answer: Option A The Kyoto Protocol was adopted in Kyoto, Japan, on 11 December 1997 and entered into force on 16 February 2005. The Kyoto Protocol was an international treaty which extended the 1992 United Nations Framework Convention on Climate Change that commits state parties to reduce greenhouse gas emissions, based on the scientific consensus that global warming is occurring and that human-made CO₂ emissions are driving it.
  • 117. Which of the following committee/s is/are associated with poverty estimation in India? [A] Kothari Committee of 1974 [B] Narasimham Committee of 1991 [C] Alagh Committee of 1979 [D] Goswami Committee of 1990 [E] None of the above Q.65
  • 118. Answer: Option C Explanation: Alagh Committee was a task force constituted by the Planning Commission under the chairmanship of YK Alagh. It recommended and constructed a poverty line for rural and urban areas on the basis of nutritional requirements and related consumption expenditure. Kothari Committee – UPSC Reforms Narsimham Committee – Banking Reforms Alagh Committee – Poverty Estimation Goswami Committee – Electoral Reforms
  • 119. No Poverty is a part of which Sustainable Development Goal (SDG)? [A] SDG 5 [B] SDG 6 [C] SDG 7 [D] SDG 13 [E] SDG 1 Q.66
  • 121. In India, The Child Sex Ratio is calculated based on one of the following age groups? [A] 0-6 years [B] 1-5 years [C] 0-4 years [D] 0-3 years [E] 1-3 years Q.67
  • 122. Answer: Option A Explanation: Sex ratio is calculated as the number of females per thousand males. The census of child sex ratio in the age group of zero to six years is conducted every ten years in India. As per 2011 Census, child sex ratio has shown a decline from 927 females per thousand males in 2001 to 919 females per thousand males in 2011.
  • 123. Which of the following Article of the Indian Constitution prohibits employment of children in factories? [A] 23 [B] 22 [C] 24 [D] 21 [E] 26 Q.68
  • 124. Answer: Option C Article 24 prohibits the employment of children below the age of 14 years in any factory, mine or other hazardous activities like construction work or railway. It is an absolute prohibition.
  • 125. How many languages are there in the Eighth Schedule of the Indian Constitution? [A] 23 [B] 24 [C] 18 [D] 22 [E] 16 Q.69
  • 126. Answer: Option D Explanation: The Eighth Schedule to the Constitution consists of the following 22 languages: Assamese, Bengali, Gujarati, Hindi, Kannada, Kashmiri, Konkani, Malayalam, Manipuri, Marathi, Nepali, Oriya, Punjabi,Sanskrit, Sindhi, Tamil, Telugu, Urdu, Bodo, Santhali, Maithili and Dogri.
  • 127. The total fertility rate (TFR) is [A] the total number of children born in a country in a given year divided by labour force [B] the number of children expected to be born per woman during her reproductive years [C] the number of births in a country divided by total population in a given year [D] the number of women aged 15-45 in a country divided by total population [E] None of the above Q.70
  • 128. Answer: Option B TFR indicates the average number of children expected to be born to a woman during her reproductive span of 15-49 years. The Total Fertility Rate (TFR) is a standard demographic indicator used internationally to estimate the average number of children that a woman would have over her childbearing years (i.e. age 15-49), based on current birth trends.
  • 129. What are the ideals that a state must keep in mind while framing policies called? A. Fundamental Rights B. Directive Principles of State Policy C. Gandhian Principles D. Socialism E. None of the above Q.71
  • 130. Answer: Option B DPSP are the ideals that must be kept in mind by the state whenever it formulates policies and enacts laws. CONSTITUTIONAL PROVISIONS Directive Principles of State Policy are enumerated in Part IV of the Constitution from Articles 36 to 51. • Constitution of Ireland (1937) was the source for the Directive principles in the Indian Constitution. • These Principles resemble the ‘Instrument of Instructions’ enumerated in the Government of India Act of 1935.
  • 131. Principles of DPSP are borrowed from which constitution? A. The USA Constitution B. Canadian Constitution C. UK Constitution D. Irish Constitution E. None of the above Answer:- [D] Irish Constitution of 1937 Q.72
  • 132. Who among the following is associated with Beej Bachao Andolan initiated in 1986? [A] Sunderlal Bahuguna [B] Pandurang Hegde [C] Medha Patkar [D] Vijay Jardhari [E] None of the above Q.73
  • 133. Answer: Option D The 'Beej Bachao Andolan', also called, 'Save the Seed Movement' was started by the farmers with the help of an activist named Vijay Jardhari in Uttarakhand in the early 1980s. It was exactly stated in 1986.
  • 134. Which article states about promotion of the educational and economic interests of SC, ST, and other the weaker sections of the people under the Directive Principles of State Policy? A. Article 46 B. Article 44 C. Article 42 D. Article 49 E. None of the above Q.74
  • 136. Which amendment made elementary education for all children a fundamental right? Also choose the article added after that: A. 80th Amendment, Article 21 B. 86th Amendment, Article 21 C. 44th Amendment, Article 22 A D. 86th Amendment, Article 21 A E. None of the above Q.75
  • 137. Answer: Option D Explanation: The 86th Amendment Act of 2002 changed the subject-matter of Article 45 and made elementary education a fundamental right under Article 21 A.
  • 138. Which part of the Indian Constitution contains the Fundamental Rights? A. Part I B. Part II C. Part III D. Part IV E. None of the above Q.76
  • 139. Answer: Option C Explanation: Article 12 to 35 contained in Part III of the Constitution deal with Fundamental Rights. These are: Right to equality, including equality before law, prohibition of discrimination on grounds of religion, race, caste, sex or place of birth, and equality of opportunity in matters of employment.
  • 140. Article 32 of the Indian Constitution guarantees: A. Right to Equality B. Right to Freedom of Religion C. Right to Constitutional Remedies D. Right against Exploitation E. None of the above Q.77
  • 141. Answer: Option C Explanation: Article 32 of the Indian Constitution gives the right to individuals to move to the Supreme Court to seek justice when they feel that their right has been 'unduly deprived'.
  • 142. The ‘melting pot’ and ‘salad bowl’ are theories related to which of the following concepts: A. Migration B. Demographic Transition C. Poverty D. Multiculturalism E. None of the above Q.78
  • 143. Answer: Option D Explanation: Multiculturalism is a society's way of dealing with cultural diversity, both at the national level and at the community level. Multiculturalism develops according to one of two theories, namely the “melting pot” theory or the “salad bowl” theory.
  • 144. Which of the following schedules provides for division of power between the Centre and States under union list, state list and concurrent list? A. 7th Schedule B. 10th Schedule C. 12th Schedule D. 1st Schedule E. None of the above Q.79
  • 145. Answer: Option A Explanation: The 7th Schedule of Indian Constitution has three lists namely, the Union list, state list, and concurrent list that show the division of power between the Union and States concerning certain subjects. The Union List has a total of 97 subjects, the State List has 66 subjects, and the Concurrent List has 47 Subjects.
  • 146. Which of the following statements correctly describes Balance of Payment Account? A. It is a systematic record of some economic transactions of residents of a country with the rest of the world during a given period of time B. It is a systematic record of all economic transactions of non-residents of a country with the residents during a given period of time C. It is a systematic record of all economic transactions of residents of a country with the rest of the world during a given period of time D. It is a systematic record of all economic transactions and non-economic transactions of residents of a country with the rest of the world during a given period of time E. None of the Above Q.80
  • 147. The balance of payments is a systematic record of all economic transactions of residents of a country with the rest of the world during a given period of time. •The balance of payments (BOP), also known as the balance of international payments, summarizes all transactions that a country’s individuals, companies, and government bodies complete with individuals, companies, and government bodies outside the country. These transactions consist of imports and exports of goods, services, and capital, as well as transfer payments, such as foreign aid and remittances. Hence Option C is the answer. Answer: Option C
  • 148. Which of the following is not accounted for in the capital account under Balance of Payment? A. Direct Investment B. External Commercial Borrowing C. Remittances and Grants D. Portfolio Investment E. All of the above are accounted for in the capital account under Balance of Payment Q.81
  • 150. There are two components of BoP- 1. Current Account 2. Capital Account Current Account deals with current, ongoing, short term transactions like trade in goods, services (invisible) etc. It reflects the nation’s net income. It has broadly 4 components. • Goods – trade in goods • Services (invisible) – trade in services e.g. tourism • Income – investment income • Current unilateral transfers – donations, gifts, grants, remittances . Note that grants might appear as the component of the capital account but are included in the current account as they are unilateral and creates no liability. The recipient does not have to give anything back in return. Option C is incorrect because remittances and grants are part of Current account. Capital Account deals with capital transactions i.e. those transactions which create assets or liabilities. It reflects the net changes in the ownership of national assets. It also has broadly 4 components • Foreign Direct Investment (FDI) • Foreign Portfolio Investment (FPI) • External Borrowings such as ECB • Reserve Account with the Central Bank. Answer: Option C
  • 151. All of the following are variables that can be manipulated to affect fiscal policy except: A. Income taxes B. Government Spending more on national highways. C. Paying higher subsidies D. Paying unemployment insurance benefits. E. Rate of interest Q.82
  • 152. Answer: Option E • The Indian government coordinates with the RBI. RBI either decreases or increases the money supply which will influence the rate of interest • Government as part of its fiscal policy tampers with Tax rates and its expenditure on various programs.
  • 153. In the context of Balance of payment (BoP), what are ‘accommodating transactions’? A. Mismatches in Balance of payment records. B. Current account transactions. C. Transactions that are done to balance the surplus or deficit of Balance of payment. D. Transactions that are done with profit maximization motive. E. None of the Above Q.83
  • 154. Accommodating Transactions •Transactions done to balance the surplus or deficit of BoP caused by the Current account and Autonomous transactions are called accommodating transactions. Hence option C is the answer. These include – •Foreign exchange reserve •Borrowing from IMF or foreign monetary authorities Answer: Option C
  • 155. Which of the following is not the correct statement relating to the benefits of small-scale industries for an economy? A. They provide immediate large-scale employment. B. They offer a method of ensuring a more equitable distribution of the national income. C. They facilitate an effective mobilisation of resources of capital and skill. D. They can compete with foreign heavy industries and can provide heavy machineries. E. They bring about the regional balances in the country. Q.84
  • 156. Answer: Option D • The Micro, Small and Medium Enterprises (MSME) sector has emerged as a highly vibrant and dynamic sector of the Indian economy over the last five decades. It contributes significantly in the economic and social development of the country by fostering entrepreneurship and generating large employment opportunities at comparatively lower capital cost, next only to agriculture. MSMEs are complementary to large industries as ancillary units and this sector contributes significantly in the inclusive industrial development of the country. The MSMEs are widening their domain across sectors of the economy, producing diverse range of products and services to meet demands of domestic as well as global markets. • As per the Annual report 2022-23 of the Ministry of Micro, Small and Medium Enterprises, the MSME sector provided employment to 11.10 Crore people. MSMEs sector contributes around 30% to India's GDP and has 40% share in the country's exports.
  • 157. _____ has recently joined the Shanghai Cooperation Organisation at its 23rd leaders’ summit, hosted virtually by India as chairman of the grouping. (a) Pakistan (b) Iran (c) Russia (d) Kazakhstan (e) Uzbekistan Q.85
  • 159. Which of the following statements are incorrect regarding BRICS? (a) BRICS does not exist in form of organization, but it is an annual summit (b) BRICS started as BRIC and Spain was invited later to join. (c) BRICS bloc of developing nations agreed to admit Saudi Arabia, Iran, Ethiopia, Egypt, Argentina and the United Arab Emirates (d) B and C (e) A and C Q.86
  • 160. Answer: Option B Announcing the decision at the end of the 15th BRICS summit at Johannesburg, Prime Minister Narendra Modi said addition of new members would strengthen the outfit and increase confidence in the idea of multipolar world order.
  • 161. Q. Which of the following ministry in India decides the rate of depreciation of a good? (a) Ministry of Finance (b) Ministry of Commerce and Industry (c) Ministry of Statistics and Programme implementation (d) Ministry of Railways (e) Ministry of defense Q.87
  • 162. Every asset (except human beings) goes for depreciation in the process of their use, which means they ‘wear and tear’. The governments of the economies decide and announce the rates by which assets depreciate (done in India by the Ministry of Commerce and Industry) and a list is published, which is used by different sections of the economy to determine the real levels of depreciation in different assets. Hence option B is the answer. Answer: Option B
  • 163. In terms of economy, the Visit by foreign nationals to witness the 2023 Cricket World Cup in India would amount to (a) Export (b) Import (c) Production (d) Consumption (e) Investment Q.88
  • 164. Exports are goods and services that are produced in one country and sold to buyers in another. The visit of foreign nationals to India amounted to export as foreign nationals will buy Indian goods and use Indian services. Also, India will get foreign currency. Hence Option A is the answer. An import is a good or service bought in one country that was produced in another. Suppose a citizen of the US buys goods and services in India, then it would amount to Import for the US. Because its citizen is buying goods and services from another country. Production - The process of making or growing goods to be sold. Consumption - The amount used or eaten. Investment - An investment is an asset or item acquired with the goal of generating income. Answer: Option A
  • 165. Q. The dominant objective of Monetary Policy is (a) Controlling Money Supply (b) Controlling Demand for Money (c) Achieving Price Stability (d) Promoting Foreign Trade (e) Reduction of Unemployment Q.89
  • 166. Explanation: Monetary Policy refers to the use of monetary instruments under the control of the central bank to influence variables, such as interest rates, money supply, and availability of credit, to achieve the objectives of the policy. Monetary Policy could have either a single objective of price stability or multiple objectives of the policy. In theory and practice, price stability is considered the dominant objective of monetary policy. For countries, which have adopted an inflation-targeting framework price stability is the core objective. Therefore, the correct answer is 'Achieving Price stability'. Hence option C is the correct answer. Answer: Option C
  • 167. Q. Which of the following statement is NOT correct regarding the New Development Bank (NDB)? (a) It is established by the BRICS countries (b) It is headquartered at Sanghai, China (c) K.V Kamath was the first president of the Bank (d) The initial authorised capital of the bank was $100 bn (e) It cannot lend to countries other than BRICS members Answer: E Q.90
  • 168. Q. It is a type of poverty generally caused by a sudden crisis or loss and is often temporary. It can be caused by environmental disasters, divorce, or severe health problems. Which kind of poverty are we referring to? (a) Generational poverty (b) Situational poverty (c) Absolute poverty (d) Relative poverty (e) None of the above Answer: B Q.91
  • 169. Q. When we say that a person has more money in his hand to save or to consume when compared to the previous year for example, then there is a definite increase in which of the following? (a) Portion of National Income received by individuals (b) Personal Disposable Income of that person (c) Private Income (d) Personal Income (e) None of the above Answer: B Q.92
  • 170. Q. Which of the prices comes into play during the billing process, where the indirect taxes levied by the government are added and this is the final cost the consumer has to pay for the product and the subsidies are subtracted because it reduces the cost of production and thus the consumer has to pay less. (a) Factor Cost (b) Basic Price (c) Market Prices (d) Current Prices (e) None of the above Answer: C Q.93
  • 171. Q. A country wants to know whether the quantity of goods produced in that country in a year has increased when compared to the previous year. Which of the following would give an idea regarding the same? (a) Real GDP (b) Nominal GDP (c) Per Capita GDP (d) Purchasing Power Parity (e) None of the Above Q.94
  • 172. GDP can be calculated at: a. Current prices (Nominal GDP) b. Constant prices (Real GDP) Nominal GDP: Nominal GDP refers to the current year's production of final goods and services valued at current year's prices. Real GDP: If this measure is adjusted for inflation; it is expressed in real terms. Real GDP refers to the current year production of goods and services valued at base year prices. Base year prices are constant prices. Answer: Option A
  • 173. Which one of the following is not a revenue receipt in government budget? A. Direct taxes B. Recovery of loans C. Profits and Dividends D. Fees, Penalty and Fine E. Interest receipts Q.95
  • 175. Which one of the following is a capital receipt in government budget? A. Interest receipts on loans given by the government to other parties B. Dividends and profits from public sector undertakings C. Borrowing of the government from public D. Income tax receipts E. Fines Q.96
  • 176. Answer: Option C • Capital receipts are receipts that create liabilities or reduce financial assets. They also refer to incoming cash flows. • Capital receipts can be both non-debt and debt receipts. • Loans from the general public, foreign governments and the Reserve Bank of India (RBI) form a crucial part of capital receipts.
  • 177. Which of the following statement(s) is/are correct regarding capital goods? 1. These are final goods and not the intermediate goods. 2. They themselves do not get transformed in the production process. 3. Over a period of time, the monetary value of most of the capital goods gets increased. 4. Capital goods are one of the factors of production. A. 1 and 3 only B. 2 and 3 only C. 1, 2 and 4 only D. 1, 2 and 3 only E. All of the above Q.97
  • 178. Answer: Option C • Capital goods are tangible assets such as buildings, machinery, equipment, vehicles and tools that an organization uses to produce goods or services in order to produce consumer goods and goods for other businesses. • Manufacturers of automobiles, aircraft, and machinery fall within the capital goods sector because their products are used by companies involved in manufacturing, shipping and providing other services. • Statement 3 is incorrect as during the course of time, the value of the capital goods gets decreased. • This concept is known as depreciation. • Depreciation: The capital gets consumed during the year due to wear and tear. This wear and tear is called depreciation. E.g. We buy a new car for Rs.5,00,000. As the years pass by, the car gets used and needs maintenance. So after say for 5 years, the value of the car does not remain the same. • The other factors of production are Land, Labour and Entrepreneurship.
  • 179. Which of the following isare not the Qualitative Tools of Monetary Policy? A. Credit Rationing B. Moral Suasion C. Cash reserve Ratio D. Direct Action E. All of the Above are the Qualitative Tools of Monetary Policy Q.98
  • 180. Answer: Option C • Qualitative tools are direct and specific in nature. Qualitative tools include persuasion by the Central bank in order to make commercial banks discourage or encourage lending which is done through moral suasion. • Qualitative also called selective credit control instruments work through regulation of margin requirement, credit rationing, regulation of consumer credit, moral suasion and direct action.
  • 181. Standing Deposit is a tool used as part of the Liquidity Adjustment Facility. Which of the following statements is wrong for SDF? A. Standing Deposit Facility can be used to suck liquidity B. Standing Deposit Facility need Government Securities​ C. Standing Deposit Facility changes with Repo Rate D. Under Standing Deposit Facility, banks can deposit money with RBI E. None of the Above Q.99
  • 183. In the context of any county, which one of the following would be considered as part of its social capital? A. The population of the country B. The stock of its buildings, other infrastructure and machines C. The size of population in the working age group D. The level of mutual trust and harmony in the society E. None of the above Q.100
  • 184. Answer: Option D • The level of mutual trust and harmony in the society. • The stock of its buildings, other infrastructure, and machines implies a physical capital. • Social capital is the values, beliefs, and attitudes that govern the nature of social interactions. • Social capital refers to connections among individuals — social networks and the norms of reciprocity and trustworthiness that arise from them i.e. the level of mutual trust and harmony in the society.